Sei sulla pagina 1di 118

ANTOLOGIA DE

ESTADISTICA

1
UNIDAD I

2
Definición de Estadística

La Estadística es la parte de las Matemáticas que se encarga del estudio de


una determinada característica en una población, recogiendo los datos,
organizándolos en tablas, representándolos gráficamente y analizándolos para
sacar conclusiones de dicha población.

Según se haga el estudio sobre todos los elementos de la población o sobre un


grupo de ella, vamos a diferenciar dos tipos de Estadística:

Estadística descriptiva. Realiza el estudio sobre la población completa,


observando una característica de la misma y calculando unos parámetros que
den información global de toda la población.

Estadística inferencial. Realiza el estudio descriptivo sobre un subconjunto de


la población llamado muestra y, posteriormente, extiende los resultados
obtenidos a toda la población.

POBLACION Y MUESTRA

Cuando se realiza un estudio de investigación, se pretende


generalmente inferir o generalizar resultados de una muestra a una población.
Este proceso de inferencia se efectúa por medio de métodos estadísticos
basados en la probabilidad.

 Población:

Representa el conjunto de todos los individuos que deseamos estudiar y


generalmente suele ser inaccesible. Tienen en común alguna característica
observable y del que se pretende obtener una serie de conclusiones.

 Muestra:

Es el conjunto menor de individuos accesible y limitado de la población


sobre el que realizamos las mediciones o el experimento con la idea de obtener
conclusiones generalizables a la población. El individuo es cada uno de los

3
componentes de la población y la muestra. Al número de individuos que forman
la muestra se llama tamaño muestral (n). La muestra debe ser representativa
de la población y con ello queremos decir que cualquier individuo de la
población en estudio debe haber tenido la misma probabilidad de ser elegido.

Las razones para estudiar muestras en lugar de poblaciones son diversas y


entre ellas podemos señalar:

o Ahorrar tiempo.
o Ahorrar costes.
o Estudiar la totalidad de los pacientes o personas con una
característica determinada en muchas ocasiones puede ser una tarea
inaccesible o imposible de realizar.
o Aumentar la calidad del estudio.
o La selección de muestras específicas nos permitirá reducir la
heterogeneidad de una población al indicar los criterios de inclusión
y/o exclusión.

 Muestreo:

El proceso de selección de los individuos se puede realizar mediante


distintas técnicas de muestreo:

- Muestreo aleatorio simple  Cada individuo tiene las mismas


posibilidades de ser elegido para formar parte de la muestra.
- Muestreo aleatorio estratificado  Aseguras que la muestra tenga la
misma proporción de una(s) variables que la población de la que
procede.
- Muestreo sistemático  El proceso de selección se basa en alguna
regla sistemática simple, por ejemplo, elegir uno de cada “n” individuos.
- Muestreo por etapas  La selección se realiza en 2 ó más etapas
sucesivas o dependientes.

VARIABLES

Lo que estudiamos en cada individuo de la muestra son las variables


(edad, sexo, peso, talla, tensión arterial sistólica, etc). Los datos son los valores
que toma la variable en cada caso. Lo que vamos a realizar es medir, es decir,
asignar valores a las variables incluidas en el estudio. Deberemos además
concretar la escala de medida que aplicaremos a cada variable.

Un consejo: “Recoger tantas variables como sean necesarias y tan


pocas como sea posible”.

Una variable es una característica observable que se desea estudiar en


una muestra de individuos, pudiendo tomar diferentes valores. Las
clasificaremos en dos tipos: variables cuantitativas o variables cualitativas.

4
a. Variables cuantitativas.  Son las variables que pueden medirse,
cuantificarse o expresarse numéricamente. Las variables
cuantitativas pueden ser de dos tipos:
o Variables cuantitativas continuas, si admiten tomar
cualquier valor dentro de un rango numérico determinado
(edad, peso, talla).
o Variables cuantitativas discretas, si no admiten todos los
valores intermedios en un rango. Suelen tomar solamente
valores enteros (número de hijos, número de partos,
número de hermanos, etc).

b. Variables cualitativas.  Este tipo de variables representan una


cualidad o atributo que clasifica a cada caso en una de varias
categorías.
o Dicotómicas (escalas nominales): La situación más sencilla
es aquella en la que se clasifica cada caso en uno de dos
grupos (hombre/mujer, enfermo/sano, fumador/no
fumador).
o Ordinal (escalas ordinales): Se requiere de un mayor
número de categorías (color de los ojos, grupo sanguíneo,
profesión, etc).

5
Ejercicios

Clasifica las siguientes variables en cuantitativas (discretas d, continuas


q) cualitativas (nominales n, ordinales o jerarquizables o)

Estados de la República Mexicana Discreto

Número de hijos de una familia Discreto

Grado militar y/o policial Ordinal o jerarquizable

Puestos por orden en la llegada de Ordinal o jerarquizable


una carrera

Nombres de los Partidos Políticos Nominales

Temperatura de los pacientes de un Continua


hospital

Edad de los estudiantes de 5to Continua


semestre

Estatura de los alumnos del equipo Continua


de basketball

Sexo de la población del Estado de Nominal


Puebla

Estado civil Nominal

Tiempo en horas Continua

6
Tarea: Investigar 5 variables de cada tipo.

Selección de la muestra de una población

Ya definimos conceptos de Universo, Población y Muestra; ahora veamos


como seleccionar correctamente la muestra.

El muestreo es el proceso de selección y extracción de una muestra a partir de


una población. Es esencial identificar la población que estará presente en el
estudio.

La selección correcta de la muestra, implica que los elementos que la


conformen representen a la población con mayor fidelidad. Existen técnicas
para la selección de muestras y son conocidas como técnicas de muestreo.

Tipos de muestreo

Muestreo probabilístico

Son de selección azarosa o aleatoria, tienen el mayor rigor científico, todos los
elementos de la población pueden ser escogidos para formar la muestra. Se
divide en:

 Muestreo aleatorio simple: se extraen al azar un número determinado de


elementos, la muestra quedará formada por los “n” elementos obtenidos
por sorteo. Las herramientas más utilizadas para la extracción de los
elementos son las tablas de números aleatorios, o bien una calculadora
con la función random

7
Este procedimiento, tiene poca o nula utilidad práctica cuando la
población que estamos manejando es muy grande.

Ejemplo: para escoger una muestra de un grupo de 60 alumnos usamos


la fórmula para el tamaño de la muestra, considerando un índice de error
e=10%

Donde: n = ____U___ = ______60 _____ ≈ 38

1+Ne2 1+(60)(0.1)2

Una vez que se determina el tamaño de la muestra, podemos numerar


esferas del 1 al 60 y escoger aleatoriamente 38 de ellas, si se toma la
esfera 47, el alumno con ese número será estudiado.

 Muestreo aleatorio sistemático: tiene el mismo carácter azaroso que el


anterior, sin embargo acá se trabaja un proceso seriado y con un solo
número y a partir de este, a intervalos constantes, se eligen los demás
hasta completar la muestra.

Sea N el tamaño de una población y n el tamaño de la muestra que


deseamos elegir.

Sea k=N/n y sea h un número al azar entre los k primeros de una lista
de todos los elementos poblacionales.

Un muestreo sistemático de n elementos consiste en seleccionar la


muestra formada por los elementos h, h+k, h+2k, . . . , h+(n-1)k.

Debemos tener en cuenta que esta muestra depende de los valores h y


k. Dada una población y un tamaño de muestra, k es un valor fijo que
indica la separación entre los elementos sucesivos de la muestra en la

8
población y que permite obtener la muestra del tamaño deseado. El
valor h se debe elegir aleatoriamente e indica el punto de inicio para
seleccionar los elementos de la muestra. Veamos unos ejemplos para
entenderlo mejor.

Ejemplo

Se acercan las Navidades y cierta empresa de turrones cree que no va a


poder entregar todos los pedidos a tiempo, a no ser que aumente la
plantilla. La empresa dispone de un listado ordenado alfabéticamente de
20 personas con las mismas características para el puesto y que
actualmente están en paro. Puesto que el tiempo apremia y no es
posible hacer una entrevista para seleccionar al personal, se decide
elegir cinco trabajadores de forma aleatoria usando el muestreo
sistemático.

Tenemos que elegir 5 elementos sistemáticamente de un total de 20, por


tanto se debe elegir uno de cada k=20/5=4.

Se elige el punto de partida eligiendo un número al azar entre 1 y 4. Si


obtenemos, por ejemplo h=2, los elementos de la muestra serán 2, 2+4,
2+2·4, 2+3·4, es decir:

2, 6, 10, 14, 8.

Ejemplo 2

Supongamos que en el ejemplo anterior se desea extraer una muestra


de 6 trabajadores.

9
Como en ese caso, k=20/6=3.33, el valor se redondea por abajo (se
trunca) es decir se toma k=3. Esto se hace así, porque si
redondeáramos hacia arriba, no habría elementos suficientes en la
población para extraer la muestra.

Por ejemplo, suponiendo que hemos obtenido h=2, los elementos de la


muestra serían 2, 5, 8, 11, 14 y 17. Pero, si hubiéramos tomado k=4, los
cinco primeros elementos de la muestra hubieran sido 2, 6, 10, 14 y 18
pero no podríamos obtener el sexto ya que daría 22 y sólo teníamos una
lista de 20 trabajadores.

 Muestreo estratificado: se usa cuando la población no es homogénea.


Consiste en subdividir la población en subconjuntos llamados estratos,
de cada estrato se extrae una muestra por alguna técnica aleatoria, la
suma de muestras de cada estrato forman la muestra total m.

Por ejemplo, si se desea estudiar la opinión que tienen los alumnos de la


Institución; sobre el director, se pueden dividir el total de alumnos en
estratos, que podrían ser los grados y grupos.

 Muestreo por conglomerados (clusters) o grupos: se usan cuando los


individuos de la población constituyen grupos naturales conglomerados
(distritos, escuelas, empresas, municipios).

La población se divide en conglomerados con diferentes elementos,


luego se selecciona aleatoriamente un número de conglomerados, la
unidad muestral es el conglomerado o cluster y la selección de muestras
se aplica a éstos, no a los elementos que conforman el conglomerado.

10
 Muestreo no probabilístico: la muestra se obtiene atendiendo al criterio
del investigador, o por razones de tiempo, economía, material o
comunidad, no utiliza el azar. Puede ser:

1. Accidental o casual: usa muestras al alcance, no responden a


planificación previa, muestras con amigos, personas que caminan
en algún sitio, etc.
2. Intencional (opinativo): se escoge de modo directo a los
elementos de la población; pues se estima que facilitarán la
información necesaria.

11
12
Ejercicios Unidad 1 Estadística

1. En una encuesta realizada a 50 estudiantes, 25 coincidieron que es mejor


descargar música que comprar CD´s piratas. Si en una zona hay
aproximadamente 58,340 adictos a la música, ¿cuántos de éstos prefieren
descargar música que comprar CD´s piratas?

Solución

Hay q hacer una regla de proporcionalidad:

25 casos favorables = __x_____ casos probables

50 total de casos 58,340 total de casos

Para obtener el valor de (x) sólo es necesario despejar:

25= ____x____

50 58,340

x= (25/50) (58,340) = 29,170

Por lo tanto es probable que de los 58,740 adictos a la música, 29,170


prefieren descargar música.

13
2. En una encuesta realizada a 223 alumnos de EMS sobre su preferencia
para comprar libros de texto, 78 prefieren comprar el libro de matemáticas,
¿Cuántos libros espera vender Editorial Santillana en la zona escolar si hay 8
escuelas con 300 alumnos cada una?

Solución

Si N=300(8) = 2,400; n=223

Por proporciones se tiene que: 78_ casos favorables

223 total de casos

Entonces: 78_ = _x_

223 2,400

Despejando: x= (78)(2,400) / 223 = 839.46 = 839

14
3. Se encuestó a un grupo de alumnos de la Institución para organizar un
evento donde tocará “grupo de rock”, se predijo que asistirán unos 1,240
chavos (as), pues de 350 encuestados, 84 confirmaron asistir a dicho evento,
¿Cuántas personas conforman la población?

Solución

Si n=350 entonces 84(casos favorables) / 350 encuestados

Explica como N= 5,166

84 (casos favorables) = 1,240 encuestados

350 encuestados x (total de casos)

N= 5,166.6

(muestreo aleatorio simple)

4. En el Bachillerato “Benito Juárez” se hará un estudio para saber si los casos


de gripe se han vuelto críticos, hay 1,425 alumnos en total. ¿Cuál será la
cantidad de la muestra a estudiar, teniendo en cuenta que se desea:

15
a. 10% error (90% confianza)
b. 5% error (95% confianza)
c. 1% error (99% confianza)
d. 0% error (100% confianza)

a) Si N= 1,425 e=0.1, entonces n= 1,425 / 1 + (1,425) (0.1) 2 = 93.44 = 93

Es decir, cuando se quiere un error de 10% en los datos, basta con tener una
muestra de 93 alumnos.

b) Si N= 1,425 e=0.05, entonces n= 1,425 / 1 + (1,425) (0.05) 2 = 312.32 = 312

Es decir, cuando se quiere un error de 5% en los datos, basta con tener una
muestra de 312 alumnos.

Realiza los incisos c y d

5. De una población de 600 alumnos, se seleccionaron 24 como muestra, para


determinar a los encuestados, se consiguieron listas en los grupos (población)
y se enumeraron del 1 al 600, luego se seleccionó de forma aleatoria 24
números (este proceso se puede hacer de la siguiente manera):

1) En una caja o bolsa transparente colocar los números del 0 al 9, extraer


uno de ellos, anotarlo y regresarlo a la urna, extraer otro, anotarlo y
regresarlo a la urna, extraer el último, anotarlo y regresarlo a la urna; así
todos los números (del 0 al 9) tienen la misma oportunidad de salir y
formar desde el 001, hasta el 999.

16
Elvira realizó este tipo de muestreo y obtuvo los siguientes números:

328 624 392 341 131 312 097 936

104 920 083 287 690 914 041 718

211 009 174 621 780 616 321 197

Observa que los números subrayados son mayores que la población,


estos se pueden modificar sin perder su sentido aleatorio:

246 209 216 069 149 369 187 166

Una vez que se ha determinado los 24 que conformarán la muestra, se


buscarán esos números en la lista, y serán a esos alumnos a quienes se
entrevistarán sobre la bebida que prefieren.

2) Usar una calculadora con la función Ran# para obtener números


aleatorios de la población. Así 600-> Ran# -> = -> presionar la tecla
cuantas veces como sea el tamaño de la muestra (24 veces en este
caso).

6. Otros proceso de muestreo.

17
1) Muestreo aleatorio sistémico: donde s=600/24=25, si escogiera al 5
como número bse, la muestra estaría compuesta por: 5, 5 + 25, 30,
25….
2) Muestreo estratificado: obtener un alumno aleatoriamente de cada
grupo en particular.
3) Muestreo por conglomerado: obtener aleatoriamente un grupo y escoger
a 24 alumnos de dicho grupo.

Actividades portafolio evidencias:

1. A partir de cada situación define la población de estudio y señala si es finita,


infinita o hipotética:

Caso Poblaci Finita Infinita Hipotétic


ón en a
estudio

Un astrónomo desea estudiar la


intensidad de las estrellas.

El director escolar dará informes


sobre el aprovechamiento
académico de la escuela.

Se producen 10 pantalones y se
someten a pruebas para ver su
resistencia y duración, una vez
satisfechas, las pruebas se lanza
al mercado a gran escala.

El presidente municipal otorgará


becas a estudiantes destacados
y de pocos recursos económicos.

18
Tito desea saber quien de los
habitantes de su pequeña
población, no paga el servicio de
agua potable.

Paula desea saber si su cafetería


tendrá éxito ya que la abrirá en el
km. 45 de la Autopista Puebla –
México

2. En los siguientes planteamientos, determina el valor de N, n o e de acuerdo


a datos proporcionados.

Famsa abrirá una sucursal en Teziutlán. Se encuesta a la población antes de


abrirla. Si en Teziutlán hay 9,400 posibles compradores:

a) ¿Cuál es el tamaño de la muestra que se requiere para obtener un 90% de


confianza?

b) ¿Y si realiza con un error del 7%, cuánto vale n?

19
c) ¿De qué tamaño sería la muestra si e=0.05?

3. Usa una fórmula para calcular n con base a la población N. Anota tus
conclusiones y toma el valor de e=5% para todos los casos.

a) N=100 b) N=780

20
c) N= 25,340 d) N=3,428,326

4. La señora de la tiendita decidió vender almuerzos completos pues espera


que se vendan muy bien ya que obtuvo un 96% de confianza de un total de 288
alumnos

a) ¿Cuál es el total de alumnos que encuestó?

21
b) Si hubiese usado el 1% de error, ¿cuántos chicos hubiera encuestado?

c) Si sólo quisiera un 40% de confianza, ¿a cuántos alumnos tendría que haber


encuestado?

5. En las siguientes afirmaciones y preguntas, propón variables asociadas al


estudio y escríbelas:

a) La empresa Mac sacará al mercado una nueva versión del iPad, ¿qué
variables a debe estudiar para que su lanzamiento sea exitoso????

22
b) La empresa HP fabrica mini-laps, algunas variables a considerar para que
sus productos sean de calidad son:

d) HBO lanzará una nueva serie juvenil, menciona algunas variables que puede
tomar en cuenta para concretar su proyecto:

e) SEDESOL otorgará un crédito a la palabra de la mujer, ¿qué variables


considera para elegir a las mujeres acreedoras a este?

6. Señala que tipo de variable corresponde a cada una de las siguientes


(cuantitativas o cualitativas).

23
Satisfacción del aula de medios de tu
Institución.

Ingresos monetarios de tu familia a la


semana.

Gramos de azufre que se queman


durante el proceso de combustión.

Tiempo que tardas en darte un


regaderazo.

Postre favorito.

7. Resuelve los siguientes ejercicios.

a) El DIF en el Estado de Puebla, realizará un estudio sobre la influencia de los


medios de comunicación en el desarrollo académico de los estudiantes, gracias
a una computadora, eligen aleatoriamente 4 escuelas (un kínder, una primaria,
una secundaria y un bachillerato), por cada escuela escogen a 10 alumnos de
cada grupo como la muestra a estudiar. ¿Cuántos tipos de muestreo se vieron
involucrados en este estudio?

b) Entrevistaron a 14 niños del Bachillerato y 8 afirmaron que durante la


primaria, veían Rugrats, ¿cuántos niños probablemente hicieron lo mismo en
esa escuela donde asisten 300 alumnos?

24
c) Un paquete de cartón de Bachoco contiene 24 huevos, uno se rompió
durante su transporte, si en un camión se espera que se rompan 300, ¿cuántos
huevos se transportaron en total?

e) El Estado de Puebla, espera que en el 2014 se gradúen 8,000 alumnos de


los inscritos en Nivel Media Superior, de los 13,000 inscritos, ya que 68
alumnos se gradúan de una muestra de….

Producto Final 1

Realiza un trabajo de campo:

1. Elige un fenómeno, producto, gusto de tu interés.


2. Define una población (N).
3. Calcula el tamaño de la muestra y explica tu método de muestreo.
4. Determina la variable a estudiar y su clasificación.
5. Menciona la importancia de las variables en Estadística.
6. Expón los puntos anteriores con el grupo y discute diversas
cuestiones con tu docente y grupo.

25
ESTADÍSTICA DESCRIPTIVA

1. Introducción

La Estadística descriptiva es la rama de las matemáticas que comprende la


recopilación, tabulación, análisis e interpretación de datos cuantitativos y
cualitativos, para tomar decisiones que se requieran a fin de que el
comportamiento de los datos se mantenga dentro de los parámetros de control
establecidos.

 Población (N)– Es el conjunto de todos los elementos de interés para


determinado estudio

 Parámetro – Es una característica numérica de la población, se


identifica con letras griegas (Media = µ, Desviación estándar = σ,
Proporción = π, Coeficiente de correlación = ρ)

 Muestra (n) – Es una parte de la población, debe ser representativa de


la misma.

 Estadístico – Es una característica numérica de una muestra, se


identifica con letras latinas (Media = X, Desviación estándar = s,
Proporción = p, Coeficiente de correlación = r)

La Estadística descriptiva proporciona un criterio para lograr mejoras, debido


a que sus técnicas se pueden usar para describir y comprender la variabilidad.
Por ejemplo, consideremos en una caldera de vapor la presión del combustible
alimentado y la eficiencia de la caldera, si utilizamos instrumentos de medición
con la resolución suficiente, encontraremos que existe variabilidad en esos
parámetros, y mediante el uso de técnicas estadísticas podemos realizar
mejoras para reducir la variación en rendimiento de la caldera.

Para poder obtener consecuencias y deducciones válidas de los datos de un


estadístico, es muy útil contar con información sobre los valores que se
agrupan hacia el centro y sobre que tan distanciados o dispersos estén unos
respecto a otros. Comenzaremos por definir estas medidas:

26
La estadística inferencial se refiere a la estimación de parámetros y pruebas
de hipótesis acerca de las características de la población en base a los datos
obtenidos con una muestra.

2. MEDIDAS DE TENDENCIA CENTRAL Y DISPERSIÓN PARA


DATOS SIMPLES.

Medidas de tendencia central

 Media: ( x̄ ) Es el promedio aritmético de todos los valores que componen


el conjunto de datos. Se calcula mediante la siguiente fórmula:
Para una muestra y para una población se tiene respectivamente:

xi xi
x̄=∑ μ=∑
n n

Ejemplo 1: En un equipo de fútbol, una muestra de estaturas de sus


integrantes son las siguientes:

1.70,1.79,1.73,1.67,1.60,1.65,1.79,1.84,1.67,1.82, 1.74. Calcule la media.

xi 19
x̄=∑ = =1 .73
n 11

~
 Mediana: ( x ) Los datos de "n" observaciones son ordenados del más
pequeño al más grande, Si el tamaño de la muestra es "non" la mediana es
el valor ordenado en la posición (n+1)/2,
Cuando el tamaño de la muestra es "par" la mediana es el promedio de los
dos valores que se encuentran al centro del conjunto de valores. Se puede
calcular mediante:

( n/2 )+ ( [ n/2 ] + 1 )
2

Ejemplo 2: Para el ejemplo anterior ¿cuál es la mediana?

27
Ordenando los datos de mayor a menor se obtiene:

1.60,1.65,1.67,1.67,1.70,1.73,1.74,1.79,1.79,1.82,1.84;

Como tenemos 11 datos el número es non por lo que (n+1)/2 = 12/2 = 6,


buscando el número que ocupa la sexta posición en los datos ordenados
~x=1. 73
encontramos el valor de la mediana

 Media acotada (Truncated Mean): Determinado porcentaje de los valores


más altos y bajos de un conjunto dado de datos son eliminados (tomando
números enteros), para los valores restantes se calcula la media.

Ejemplo 3: Para la siguiente serie de datos calcule la media acotada al


20%:

68.7,34.3,97.9,73.4,8.4,42.5,87.9,31.1,33.2,97.7,72.3,54.2,80.6,71.6,82.2,

Como tenemos 11 datos, el 20% de 11 es 2.2, por lo cual eliminamos 2


datos el más bajo y el más alto, ordenado los datos obtenemos:

8.4,31.1,33.2,34.3,42.5,54.2,68.7,71.6,72.3,73.4,80.6,82.2,87.9,97.7,97.9,
los valores a eliminar son: 8.4 y 97.9; calculando la media de los datos
(~x , .20 )=63. 82
restantes obtenemos

Medidas de dispersión

Para comprender el concepto de varianza, supóngase que tenemos los datos


siguientes de los cuales queremos saber que tan dispersos están respecto a su
media:

28
2, 3, 4, 5, 6 con media = 20/5 = 4

Si tomamos la suma de diferencias de cada valor respecto a su media y las


sumamos se tiene:

(-2) + (-1) + (0) + (1) +(2) = 0

Por lo que tomando diferencias simples no es posible determinar la dispersión


de los datos.

Si ahora tomamos esas mismas diferencias al cuadrado y las sumamos se


tiene:

4 + 1 + 0 + 1 + 4 = 10

Varianza de los datos

Es una medida que nos ayuda a comprender la variabilidad de los datos, que
tan distanciados están de la media

 Poblacional (σ2 ) Se obtiene dividiendo el valor anterior entre n = 5, o


sea el promedio de la suma de las diferencias al cuadrado, tomando n
datos.

2 ( xi− x̄ )2
σ =∑
n

 Poblacional (s2 ) Se obtiene dividiendo el valor anterior entre n - 1 = 4, o


sea el promedio de la suma de las diferencias al cuadrado, tomando n -1
datos.

29
2
2 ( xi− x̄ )
s =∑
n−1

 Desviación estándar: Es la raíz cuadrada de la varianza:

(xi− x̄ )2
Para el caso de una población
σ=
√ ∑ n

( xi− x̄ )2
Para el caso de una muestra
s=
√ ∑ n−1

 Rango ( R ): es la diferencia positiva entre el valor mayor y el valor


menor de un conjunto de datos. Por ejemplo para el conjunto de datos
siguiente:
2.0,2.1,2.4,2.5,2.6,2.8,2.9,2.9,3.0,3.1,3.6,3.8,4.0,4.0

Su rango es R = 4.0 – 2.0 = 2.0

 Coeficiente de Variación (CV): Se utiliza para comparar la dispersión


de dos conjuntos de datos que tienen unidades diferentes, ya que
representa una medida relativa de dispersión.
s
Coeficiente. de . var iación=CV = (100)

Por ejemplo si la media de tiempos de respuesta es de 78.7 y su desviación


estándar es 12.14, el CVt:

12. 14
CV t = (100 )=12 . 05 %
78. 7
Por otra parte si la media de temperaturas es de 10 y su desviación estándar
de 2, el CVs de las temperaturas es:

30
2
CV s = (100 )=20 %
10

Por tanto la dispersión de las temperaturas es mayor que la de los tiempos de


de respuesta, es posible comparar estas dispersiones con el CV aunque los
dos conjuntos de datos sean completamente disímbolos.

Ejemplo 4: La resistencia al rompimiento de dos muestras de botellas es la


siguiente:

Muestra 1: 230 250 245 258 265 240


Muestra 2: 190 228 305 240 265 260

Calcule la desviación estándar para ambas muestras.

Muestra 1: Muestra 2

⃗x =248 ⃗x =248

Suma(Xi - x̄ )2 = 790 Suma(Xi - x̄ )2 = 7510

n-1=5 n-1 = 5

790 7510
s= √ 5 = 12.56 s= √ 5 = 38.75

Rango = 265 – 230 = 35 Rango = 305 – 190 = 115

31
CV = 12.56/248*100= 5.06% CV = 38.75/248*100 = 15.625

Aunque la media en ambas muestras es la misma, la desviación estándar (s),


rango y coeficiente de variación, son menores en la muestra 1, por lo cual
deducimos que es presenta menor variabilidad.

Ejemplo 5:

Se desea hacer un estudio estadístico de la temperatura del agua, para esto es


necesario tomar una muestra y calcular la media, mediana, media acotada al
15%, desviación estándar, rango y coeficiente de variación. Se realizan 14
observaciones arrojando los siguientes resultados en ºC: 2.11, 3.8, 4.0, 4.0,
3.1, 2.9, 2.5, 3.6, 2.0, 2.4, 2.8, 2.6,2.9, 3.0.

1) Calcular la media, mediana, desviación estándar, media acotada al 5%,


desviación estándar, rango y coeficiente de variación.

3. OTRAS MEDIDAS DE DISPERSIÓN: PERCENTILES, DECILES


Y QUARTILES

Cada conjunto de datos ordenado tiene tres cuartiles que lo dividen en cuatro
partes iguales. El primer cuartil es ese valor debajo del cual clasifica el 25% de
las observaciones y sobre el cual se encuentra el 75% restante. El segundo
cuartil divide a los datos a la mitad similar a la mediana.

Los deciles separan un conjunto de datos ordenado en 10 subconjuntos iguales


y los percentiles en 100 partes, la ubicación de un percentil se encuentra en:

P
L p =(n+1 )
100

Donde:

32
Lp es el sitio del percentil deseado en una serie ordenada

n es el número de observaciones

P es el percentil deseado

Por ejemplo para el conjunto de datos siguiente:

3 10 19 27 34 38 48 56 67 74

4 12 20 29 34 39 48 59 67 74

7 14 21 31 36 43 52 62 69 76

9 15 25 31 37 45 53 63 72 79

10 17 27 34 38 47 56 64 73 80

La localización del percentil 35 se halla en:

35
L35=(50+1 ) =17 . 85
100

O sea que el percentil 35 está al 85% del trayecto comprendido entre la


observación 17 que es 29 y la observación 18 que es 31 o sea L35 = 29 +
(0.85)(31-29) = 30.7. Por tanto el 35% de las observaciones están por debajo
de 30.7 y el 65% restante por encima de 30.7.

De la misma forma los percentiles 25, 50 y 75 proporcionan la localización de


los cuartiles Q1, Q2 y Q3 respectivamente.

 Q1: es el número que representa al percentil 25 (hay 25% de los datos


por debajo de este).

 Q2 o Mediana: es el número que representa al percentil 50 (hay 50% de


los datos por debajo de este).

33
 Q3: es el número que representa al percentil 75 (hay 75% de los datos
por debajo de este).

 Rango o Recorrido intercuartílico: es la diferencia entre Q1 y Q3.

DIAGRAMA DE CAJA

Es la representación gráfica de los datos en forma de caja:

1 10 4

1050
Weight

950
Q3 + 1.5 RIC

Q3 850

Q2 Mediana

Q1

Q1 – 1.5RIC

Rango
Intercuartílico
= RIC = Q3 – Valores
Q1 atípicos Bigotes

4. DISTRIBUCIÓN DE FRECUENCIAS E HISTOGRAMAS

Cuando tenemos una cantidad grande de datos es difícil poder analizarlos, a


menos que hagamos uso de herramientas que nos permitan hacerlo con mayor
facilidad y claridad. El histograma es una de ellas, consiste en un diagrama de
barras donde las bases corresponden a los intervalos y las alturas a las

34
frecuencias. Para construir un histograma es necesario tener un mínimo de 50
a 100 datos. Se tienen las siguientes definiciones:

 Distribución de frecuencias: es un resumen tabular de un conjunto de


datos que muestra el número o frecuencia de artículos en cada una de
varias clases que no se traslapan.

 Frecuencia relativa (f): Es la frecuencia de la clase dividida entre el


total n de datos. Se puede representar en porcentaje.

 Distribución de frecuencias porcentuales: es la representación de las


frecuencias relativas porcentuales.

 Frecuencia acumulada (F): es la acumulación secuencial de las


frecuencias de cada clase.

Ejemplo 6

Construir un histograma con la siguiente serie de datos:

2.41 17.87 33.51 38.65 45.70 49.36 55.08 62.53 70.37 81.21

3.34 18.03 33.76 39.02 45.91 49.95 55.23 62.78 71.05 82.37

4.04 18.69 34.58 39.64 46.50 50.02 55.56 62.98 71.14 82.79

4.46 19.94 35.58 40.41 47.09 50.10 55.87 63.03 72.46 83.31

8.46 20.20 35.93 40.58 47.21 50.10 56.04 64.12 72.77 85.83

9.15 20.31 36.08 40.64 47.56 50.72 56.29 64.29 74.03 88.67

11.59 24.19 36.14 43.61 47.93 51.40 58.18 65.44 74.10 89.28

12.73 28.75 36.80 44.06 48.02 51.41 59.03 66.18 76.26 89.58

13.18 30.36 36.92 44.52 48.31 51.77 59.37 66.56 76.69 94.07

15.47 30.63 37.23 45.01 48.55 52.43 59.61 67.45 77.91 94.47

16.20 31.21 37.31 45.08 48.62 53.22 59.81 67.87 78.24 94.60

35
16.49 32.44 37.64 45.10 48.98 54.28 60.27 69.09 79.35 94.74

17.11 32.89 38.29 45.37 49.33 54.71 61.30 69.86 80.32 96.78

Paso 1: Contar el número de datos n = 130

Paso 2: Calcular el rango R = Valor mayor – Valor menor, R = 96.78-2.41 =


94.37.

Generalmente los datos no están ordenados por lo cual resulta conveniente


ordenarlos de menor a mayor para tener una mejor visualización. En el
ejemplo los datos ya han sido previamente ordenados.

Paso 3: Seleccionar el número de columnas, mediante √n =


√ 130=11.4≈11 . Por lo cual el histograma se compone de 11 columnas

Paso 4: Calcular el tamaño del intervalo de clase ( C ), dividiendo el rango


94 . 37
=8 .58≈9
entre el número de columnas: C = 11 , resultando el tamaño del
intervalo 9.

 Otra manera de calcular el tamaño del intervalo es el siguiente:


Dividir el valor del rango entre un cierto número de clases (K). La tabla de abajo
es una guía que nos muestra para diferentes cantidades de datos el número
recomendado de clases a utilizar.

Número de datos (N) Número de clases (K)

Menos de 50 5–7

50 a 100 6 – 10

100 a 250 7 – 12

Más de 250 10 – 20

36
Paso 5: Calcular los limites de clase de cada intervalo: [0-8], [ 9-17], etc.,
considerando que el tamaño del intervalo representa la diferencia entre dos
límites de clase adyacentes ya sean inferiores o superiores.

Paso 6: Contar el número de valores que caen en cada intervalo utilizando una
hoja de registro, de esta manera se obtiene la frecuencia para cada intervalo.

Tabla 1.

Columna Intervalo Registro de frecuencias

1 0 -8 IIIII 5

2 9-17 IIIII IIII 9

3 18-26 IIIII I 6

4 27-35 IIIII IIIII I 11

5 36-44 IIIII IIIII II 17

6 45-53 IIIII IIIII IIIII IIIII IIIII III 28

7 54-62 IIIII IIIII IIIII III 18

8 63-71 IIIII IIIII III 13

9 72-80 IIIII IIIII 10

10 81-89 IIIII III 8

11 90-98 IIIII 5

Histograma

30
25
20
Frecuencia

15 Frecuencia
10
5
0
9 18 27 36 45 54 63 72 81 90 99
Clase

Paso 7: Basándose en los datos anteriores construya el histograma.

37
Diagrama de tallo y hojas
Es otra representación de la información, primero se ordenan los dígitos
principales a la izquierda de una línea vertical. A la derecha de esta línea se
registra el último dígito para cada dato conforme se revisan las observaciones
en el orden en que se registraron. Por ejemplo:

Con Minitab: Stat > EDA > Steam and leaf… Indicar columna de datos,
increment = 10

Stem-and-leaf of Respuest N = 50

Leaf Unit = 1.0

2 6 89

8 7 233566

16 8 01123456

(11) 9 12224556788

23 10 002466678

14 11 2355899

7 12 4678

3 13 24

1 14 1

5. MEDIDAS DE TENDENCIA CENTRAL Y DE DISPERSIÓN PARA DATOS


AGRUPADOS.

 La media con datos agrupados: se calcula así:

X̄ g =
∑ fM
n
38
Donde

f es la frecuencia o número de observaciones en cada clase

M es el punto medio de cada clase, se determina como el valor medio entre


los límites de clase.

n es el tamaño de la muestra o la suma de todas las frecuencias de las


clases

Ejemplo:

Clase Frecuencia de clase Frecuencia


acumulada

(Presión) (días) M fM F

50-59 3 54.5 163.5 3

60-69 7 64.5 451.5 10

70-79 18 74.5 1341.0 28

80-89 12 84.5 1014.0 40

90-99 8 94.5 756.0 48

100-109 2 104.5 209.0 50

50 3935.0

3935
X̄ g = =78 . 7
50

 Mediana de datos agrupados:

39
Primero se identifica la clase donde se encuentra la mediana cuya F es >= n /
2, en este caso la clase de 70 a 79 con punto central de clase = 74.5.

n/2−F 50/2−10
Mediana=~
X =Lmd +
[ f md ]
(C )=70+
18 [ ]
10=78 . 33 pasajeros

Donde:

Lmd es el límite inferior de la clase de la mediana cuya F es >= n / 2 o sean


(70)

F es la frecuencia acumulada de la clase que antecede a la clase de la


mediana (10)

Fmd es la frecuencia de la clase de la mediana (18)

C es el intervalo de clase de la mediana que es la diferencia entre dos límites


de clase (10)

 Moda de datos agrupados:

Primero se halla la clase que tenga la frecuencia más alta, en este caso la
clase 70 a 79.

Da
Moda=Lmo +
[ Db+ D a ] (C )=70+
[ 18−7
(18−12 )+(18−7 ) ]
10=76 . 47

Donde:

Lmo es el límite inferior de la clase modal con la frecuencia más alta (70).

Da es la diferencia entre la frecuencia de la clase modal y la clase que la


antecede (18 – 7 = 11)

40
Db es la diferencia entre la frecuencia de la clase modal y la clase que le sigue
(18 – 12 = 6)

C es el intervalo de la clase modal ( 80 – 70 = 10 )

 Varianza y desviación estándar de datos agrupados:

2∑ fM 2 −n X̄ 2
s=
n−1
s= √ s2

Para los datos anteriores se tiene:

Clase Frecuencia de clase

(Presión) (días) M fM M2 fM2

50-59 3 54.5 163.5 2790.25


8910.75

60-69 7 64.5 451.5 4160.25


29121.75

70-79 18 74.5 1341.0 5550.25


99904.50

80-89 12 84.5 1014.0 7140.25


85683.00

90-99 8 94.5 756.0 8930.25


71442.00

100-109 2 104.5 209.0 10920.25


21840.50

3935.0 31690
2.50

41
3935
X̄ g = =78 . 7
50
2
2 316902. 50−50 (78 . 7 )
s= =147 . 31 pasajeros
49
s=12 .14 pasajeros

Con esta información el personal puede tomar sus decisiones

6. USOS FRECUENTES DE LA DESVIACIÓN ESTÁNDAR

 EL TEOREMA DE TCHEBYSHEV

1
(1− )%
Establece que para todo conjunto de datos por lo menos K2de las
observaciones se encuentran dentro de  K desviaciones estándar de la media,
con K >= 1.

Por ejemplo si K =  3 desviaciones estándar respecto a la media, se tiene que


por lo menos el:

1 1
(1−
K2
)%= 1−
( )
32
%=88 . 89 %

De las observaciones estarán dentro de dicho intervalo.

CASO DE LA DISTRIBUCIÓN NORMAL

68.3% de las observaciones se encuentran dentro de  1 desviación estándar


de la media

42
95.5% de las observaciones se encuentran dentro de  2 desviaciones
estándar de la media

99.7% de las observaciones se encuentran dentro de  3 desviaciones


estándar de la media

 SESGO

En la distribución normal si no es simétrica y tiene una cola más amplia del lado
derecho, se dice que existe un sesgo a la derecha y viceversa.

El coeficiente de sesgo o asimetría P se determina como sigue:

3( X̄ −Mediana)
P=
s

Si P < 0 los datos están sesgados a la izquierda, si P > 0 están sesgados a


la derecha; si P = 0 están distribuidos normalmente.

Para el caso de los datos del ejemplo anterior se tiene:

3(78 .7−78 .33 )


P= =0 . 03
12 .14 Los datos están un poco sesgados hacia la
derecha.

Coeficiente de asimetría de Fisher

43
Otra estimación del sesgo o coeficiente de asimetría se hace a través de
momentos estadísticos (diferencias contra la media) como lo sugiere Fisher:

(X i  X)j
Mj  i 1
j  1, 2,3, 4
n
n
1
∑ ( Xi− X̄ )3
n i=1
γ 1= 3 /2
M3 1
n
Sesgo= β^ 1 =
M 32 /2 o
( ∑ ( Xi− X̄ )2
n i =1 ) Para la distribución normal
debe ser 0.

Se puede considerar que una distribución es simétrica si γ 1=0 , asimétrica


hacia la izquierda con γ 1<0 o hacia la derecha γ 1>0 .

Por ejemplo:

Ejemplo de una distribución con sesgo negativo o sesgada hacia la izquierda


con Sesgo = -1.01

Ejemplo de una distribución con sesgo positivo o sesgada hacia la derecha con
Sesgo = 1.08

44
 CURTOSIS

En la distribución normal si no es acampanada y es más picuda o aplanada de


lo normal se dice que tiene una Curtosis diferente de cero que es lo normal, si
es mayor es más picuda o más plana al revés.

Coeficiente de Curtosis de Fisher


n
1
∑ ( Xi− X̄ )4
n i=1
γ 2= 2
−3
n
M 1
Kurtosis   2  42
M2 - 3 o ( ∑ ( Xi− X̄ )2
n i =1 ) Para la distribución normal
debe ser 0.

La distribución es mesocúrtica (plana normal) si γ 2=0 , leptocúrtica si


γ 2>0 más puntiaguda que la normal o platicúrtica (más plana que la
normal ) con γ 2<0 .

Ejemplo de curva más plana que la normal Curtosis = -1.03

45
Ejemplo de curva más picuda que la normal Curtosis = 0.76

7. USO DE MINITAB y EXCEL

Para la obtención de las estadísticas descriptivas con Minitab las instrucciones


son:

 Stat > Basic statistics > Display descriptive statistics

Indicar las variables de las cuales se quieren obtener las estadísticas básicas y
la variable categórica si se desean varios grupos.

46
Seleccionar las gráficas opcionales para los datos: Histograma, diagrama de
caja y de puntos.

Seleccionar los estadísticos específicos que se desean obtener:

Los resultados son los siguientes:

Descriptive Statistics: Peso en gr

Variable Línea N N* Mean SE Mean StDev Minimum Q1 Median

Peso en gr 1 250 0 3999.6 3.14 49.6 3877.0 3967.8 3999.5

2 250 0 4085.6 3.32 52.5 3954.0 4048.8 4087.0

47
Variable Línea Q3 Maximum

Peso en gr 1 4040.0 4113.0

2 4121.5 4202.0

Diagramas de caja en Minitab:

1. Capture datos en la hoja de trabajo: 7 8 9 9 11 12 12 13 14 15 16


17 18 19 20 22
2. Seleccione la opción: Graph> Boxplot
3. Seleccione la variable C1 como se muestra en la pantalla y presione clic
en ok
4. A continuación se muestra el diagrama de caja:

Boxplot of Caja
22.5

20.0

17.5

15.0
Caja

12.5

10.0

7.5

5.0

Histograma en Minitab:

1. Capture los datos del ejemplo 6 en la hoja de trabajo:


2. Seleccione la opción: Graph> Histogram (simple)
3. Seleccione la variable C1 como se muestra en la pantalla y presione
clic en ok
4. En Options se puede cambiar el número de celdas con Number of
intervals (6 – 8)
5. A continuación se muestra el Histograma:

48
Histogram of DATOS
40

Frequency 30

20

10

0
-10 20 50 80 110
DATOS

Prueba de normalidad en Minitab:

1. Capture los datos del ejemplo 6 en la hoja de trabajo:


2. Seleccione la opción: Stat > Basic statistics
3. Seleccione la variable C1 como se muestra en la pantalla y presione
clic en ok
4. Seleccione la prueba de Anderson Darling
5. A continuación se muestra la gráfica normal, si P value > 0.05 los
datos son normales.

Probability Plot of DATOS


Normal
99.9
Mean 50.05
StDev 22.50
99
N 130
AD 0.380
95 P-Value 0.399
90
80
70
Percent

60
50
40
30
20
10
5

0.1
0 30 60 90 120
DATOS

49
USO DE EXCEL

1. En el menú Herramientas seleccione la opción Análisis de datos.


Datos de ejemplo 6.
2. Seleccione la opción Estadística descriptiva.
3. Seleccione el rango de entrada, estos corresponden a los datos
numéricos de la tabla.
4. Seleccione Resumen de estadísticas.
5. En opciones de salida seleccione en Rango de salida, una celda de
la hoja de calculo que este en blanco (a partir de está celda serán
insertados los resultados).

La hoja mostrará las siguientes medidas estadísticas de los datos


presentados:

Columna1

50.053769
Media 2
Error típico 1.9738137
Mediana 49.345
Moda 50.1
Desviación 22.504938
estándar 8
Varianza de la
muestra 506.47227
-
Curtosis 0.4466339
Coeficiente de -
asimetría 0.0352296
Rango 94.37
Mínimo 2.41
Máximo 96.78
Suma 6506.99
Cuenta 130

50
EJERCICIOS:

1. Las empresas de generación de energía eléctrica están interesadas en los


hábitos de consumo de los clientes para obtener pronósticos exactos de las
demandas de energía. Una muestra de consumidores de 90 hogares con
calefacción de gas arrojó lo siguiente (FURNACE.MTW):

BTU.In_1

2.97 7.73 9.60 11.12 13.47

4.00 7.87 9.76 11.21 13.60

5.20 7.93 9.82 11.29 13.96

5.56 8.00 9.83 11.43 14.24

5.94 8.26 9.83 11.62 14.35

5.98 8.29 9.84 11.70 15.12

6.35 8.37 9.96 11.70 15.24

6.62 8.47 10.04 12.16 16.06

6.72 8.54 10.21 12.19 16.90

6.78 8.58 10.28 12.28 18.26

6.80 8.61 10.28 12.31

6.85 8.67 10.30 12.62

6.94 8.69 10.35 12.69

7.15 8.81 10.36 12.71

7.16 9.07 10.40 12.91

7.23 9.27 10.49 12.92

7.29 9.37 10.50 13.11

7.62 9.43 10.64 13.38

7.62 9.52 10.95 13.42

51
7.69 9.58 11.09 13.43

a) Determinar los estadísticos de tendencia y dispersión

b) Construir un diagrama de caja e histograma

c) Realizar una prueba de normalidad de los datos

d) Establecer conclusiones

52
UNIDAD II

53
PERMUTACIONES

Permutación: Conjunto ordenado de n elementos.

Notación: Pn ; Pn,n ; An, n

Permutación de 5 elementos

P5 = 5! Por lo que:

Pn = n!

P5 = 5! = 5 x 4 x 3 x 2 x 1 = 120

Ejemplo 1:

Para el conjunto {a, b, c} existen las siguientes permutaciones:

Solución:

Abc, acb, bca, bac, cab, cba = 6

P3 = 3! = 6

Ejemplo 2:

En una asamblea de accionistas, hay 6 personas que han solicitado hacer uso de la
palabra ¿En cuántas órdenes diferentes pueden hablar, si es que no se ha establecido
un orden de prioridades?

Solución:

P6 = 6! = 720 formas distintas

Ejemplo 3:

En un proceso de manufactura hay seis operaciones distintas, que se indican con A, B,


C, D, E y F. En general no existe una secuencia fija para las operaciones, con la
salvedad de que A debe efectuarse al principio y F al final. ¿Cuántas secuencias
diferentes pueden ocurrir?

Solución:

54
A B C D E F
P4 = 4! = 24 formas diferentes

Cuando se toman parte de los elementos del conjunto se tiene: 

n!
Pn,r = (n−r)!  

Ejemplo 4:

Si : n = 5 y r = 3 

5! 5 ! 120
= = =60
P5,3 = (5−3 )! 2! 2

Ejemplo 5:

Hay 7 candidatos para desempeñar 3 tareas, si todos los candidatos son igualmente
eficientes, ¿De cuántas maneras se puede efectuar la asignación? 

Solución:

7! 7! 7.6.5.4!
= = =210
P7,3 = (7−3)! 4! 4!

  Ejemplo 6:

De cuántas maneras 3 fresadoras, 4 tornos, 4 taladros y 2 cepillos pueden ordenarse


en fila en un taller, de modo que el mismo tipo de máquina queden juntas. 

3F 4T 4T 2C

P3 = 3! P4 = 4! P4 = 4! P2 = 2!

P4 = 4!

3! x 4! x 4! x 2! x 4! = 165,888 maneras diferentes

55
 En conclusión… Qué es una permutación?

Son maneras de distribuir elementos, se conocen también como ordenaciones pues si


un mismo elemento aparece en más de un evento pero en diferente orden o posición,
se consideran como puntos muestrales diferentes. Es decir:

 El orden si importa.
 Si entran todos los elementos,
 Pero no se repiten todos los elementos.

Hay dos tipos de permutaciones:

1. Se permite repetir: como la cerradura de arriba, podría ser "333".

2. Sin repetición: por ejemplo los tres primeros en una carrera. No puedes
quedar primero y segundo a la vez.

1. Permutaciones con repetición

Son las más fáciles de calcular. Si tienes n cosas para elegir y eliges r de ellas, las
permutaciones posibles son:

n × n × ... (r veces) = nr

(Porque hay n posibilidades para la primera elección, DESPUÉS hay n posibilidades


para la segunda elección, y así.)

Por ejemplo en la cerradura de arriba, hay 10 números para elegir (0,1,...,9) y eliges 3
de ellos:

10 × 10 × ... (3 veces) = 103 = 1000 permutaciones

56
Así que la fórmula es simplemente:

nr

donde n es el número de cosas que


puedes elegir, y eliges r de ellas
(Se puede repetir, el orden importa)

2. Permutaciones sin repetición

En este caso, se reduce el número de opciones en cada paso.

Por ejemplo, ¿cómo podrías ordenar 16 bolas de billar? Después de elegir por ejemplo
la "14" no puedes elegirla otra vez.

Así que tu primera elección tiene 16 posibilidades, y tu siguiente elección tiene 15


posibilidades, después 14, 13, etc. Y el total de permutaciones sería:

16 × 15 × 14 × 13 ... = 20,922,789,888,000

Es decir, hay 3,360 maneras diferentes de elegir 3 bolas de billar de entre 16.

¿Pero cómo lo escribimos matemáticamente? Respuesta: usamos la "función


factorial".

La función factorial (símbolo: !) significa que se multiplican números


descendentes. Ejemplos:

 4! = 4 × 3 × 2 × 1 = 24

 7! = 7 × 6 × 5 × 4 × 3 × 2 × 1 = 5040

57
 1! = 1

Nota: en general se está de acuerdo en que 0! = 1. Puede que


parezca curioso que no multiplicar ningún número dé 1, pero ayuda a
simplificar muchas ecuaciones.

Así que si quieres elegir todas las bolas de billar las permutaciones serían:

16! = 20,922,789,888,000

Pero si sólo quieres elegir 3, tienes que dejar de multiplicar después de 14. ¿Cómo lo
escribimos? Hay un buen truco... dividimos entre 13!...

16 × 15 × 14 × 13 × 12 ...

  = 16 × 15 × 14 = 3360

13 × 12 ...

¿Lo ves? 16! / 13! = 16 × 15 × 14

La fórmula se escribe:

La fórmula se escribe:

donde n es el número de cosas que


puedes elegir, y eliges r de ellas
(No se puede repetir, el orden
importa)

Ejemplos:

Nuestro "ejemplo de elegir en orden 3 bolas de 16" sería:

58
16! 16! 20,922,789,888,000

= = = 3360

(16-3)! 13! 6,227,020,800

¿De cuántas maneras se pueden dar primer y segundo premio entre 10 personas?

10! 10! 3,628,800

= = = 90

(10-2)! 8! 40,320

(que es lo mismo que: 10 × 9 = 90)

Notación

En lugar de escribir toda la fórmula, la gente usa otras notaciones como:

Ten en cuenta las siguientes notaciones:

Permutaciones (Orden sí importa) Combinación (Orden no importa)

Observa las siguientes figuras:

59
1. Dibuja las combinaciones de las figuras si se hacen grupos de 4.

a)

b)

60
c)

d)

e)

61
2. Dibuja las permutaciones de las figuras, excluyendo la estrella, es decir, grupos de
2 figuras:

a) g)

b) h)

c) i)

62
d) j)

e) k)

d) l)

2. ¿De cuántas formas distintas pueden sentarse ocho personas en una fila de
butacas?

Sí entran todos los elementos. Tienen que sentarse las 8 personas.

Sí importa el orden.

No se repiten los elementos. Una persona no se puede repetir.

63
3. ¿ En cuántas formas diferentes pueden sacarse cuatro cartas (a la vez)de un
paquete de 52 cartas?

4. Calcula el número de formas en que un ejecutivo puede elegir a 3 de 15 empleados


para un ascenso.

5. Calcula en número de formas en que un capataz puede escoger a 12 de 18


trabajadores para asignarles trabajo en tiempo extra.

6. ¿Cuántos comités de 4 personas se pueden formar con 9 personas?

7. ¿De cuántas maneras se puede escoger un comité, compuesto de 3 hombres y 2


mujeres, de un grupo de 7 hombres y 5 mujeres?

64
Combinaciones :
Una combinación de “ n ” elementos tomados de “ r ” en “ r ” es un subconjunto no
ordenado de “r” elementos con r ≤ n. 

2 combinaciones formadas por r elementos son distintas, si difieren al menos en un


elemento. 

Ejemplo 1:

Sea el conjunto {a, b, c} de cuántas maneras podemos seleccionar:

a)      un elemento

b)      dos elementos

c)      tres elementos 

Solución:

a)      Existen 3 formas de seleccionar un elementos: a; b; c.

b)      Existen 3 formas de seleccionar dos elementos: ab, ac, bc

c)      Existe 1 forma de seleccionar 3 elementos: abc 

Notación: nCr;
(nr )  

Para determinar el número de combinaciones de n elementos tomando de r en r:

n!
Crn=
r !(n−r )!

Ejemplo :

Si n = 10 r = 7 

10! 10! 10 .9.8.7! 720


C10
7 = = = = =120
7!(10−7)! 7!.3 ! 7!.3 ! 6

65
Ejemplo 2:

El congreso anglomexicano de administración pública, debe elegir el futuro comité


ejecutivo que regirá a esa institución durante el próximo año.

La comisión directiva se forma con 6 integrantes y este año han sido propuestos 7
representantes mexicanos y 4 ingleses para ser electos. Se pide determinar de
cuántas maneras se puede integrar la comisión en los siguientes casos:

a)      Si en la comisión debe haber 4 mexicanos y 2 ingleses.

b)      Si en la comisión debe haber como mínimo 2 ingleses y 2 mexicanos.

Solución:

7!
C74= =35
a)      Los mexicanos se pueden escoger de: 4!. 3!

Los ingleses se pueden escoger de:

4!
C24 = =6
2 !.2!

7
Conjuntamente : C 4 . C24 = 35 x 6 = 210

b)      Se pueden presentar los casos:

4 7
1)      2 ingleses y 4 mexicanos: C2 C 4 = 6 x 35 = 210

2)      3 ingleses y 3 mexicanos: C34 C73 = 140

3)      4 ingleses y 2 mexicanos: C 44 C73 = 21

210 + 140 + 21 = 371

Ejemplo 3:

En los laboratorios “ELKO” hay 3 plazas vacantes de un total de 33 solicitudes de


empleo, sólo 14 se han considerado aceptables, en bases en las entrevistas

66
practicadas por el departamento de personal. ¿De cuántas maneras pueden asignarse
las 3 plazas?

a)      Si todos los empleos son de la misma categoría

b)      Si un empleo es de gerente de ventas, uno es de agente visitador para las


ciudades de Puebla y Tlaxcala y otro de agente visitador para las ciudades de
Tampico y Cd. Madero.

Solución:

a) C14
3 = 364

14 !
14 P3 = =2184
b)      11!

Problemas y Ejercicios Propuestos:

1. De 7 candidatos ¿ Cuántas ternos se pueden escoger?

2. 4 alumnos deciden el horario en el cual harán sus prácticas pre profesionales ,


sabiendo que existen 6 turnos disponible distintos y en cada turno debe asistir uno de
ellos. Entonces cuántas formas pueden practicar

3. En la distribución de material para médico para 5 hospitales se tomó en cuenta lo


siguiente :

A uno de ellos se debe entregar el material a las 8:30 al otro a las 9:00 y al
siguiente a las 9:30 y así sucesivamente hasta el último . Si existe la posibilidad de
variar el orden de entrega de material a cada hospital, entonces de cuantas formas
distintas se entrega el material.

67
4. Cuántos objetos distintos deben existir para que el número de combinación que se
puede formar , tomándolos de 2 en 2 sea igual a 6 veces el número de objetos

5. Calcular :

35 !*28!
a )K =
27 !*36 !
29 !
b )K =
27 !+28 !
36 !+37 !
c ) K=
37 !−36 !

SESION 02

COMBINACIONES

Análisis Combinatorio

Principio Fundamental del Conteo:

Suponga que una persona tiene 2 formas de ir de una ciudad A a otra ciudad B; y una
vez llegada a B, tiene 3 maneras de llegar a otra ciudad C, ¿De cuántas maneras podrá
realizar el viaje de A a C pasando por B?

Si empezó a pie, podrá tomar luego avión, carro o trasatlántico, y si empezó en


bicicleta, también podrá tomar avión, carro o trasatlántico.

68
La persona tuvo 6 formas diferentes de realizar el viaje que son:

(iniciales) pa, pc, pt, ba, bc, bt. (2 x 3 = 6)

Se puede representar en un diagrama de árbol

Por lo que el principio fundamental del análisis combinatorio, puede expresarse así:

Si una primera decisión, operación o acción puede efectuarse de a formas diferentes,


una segunda acción puede efectuarse de b formas diferentes, una tercera acción
puede efectuarse de c formas diferentes y así sucesivamente hasta la enésima acción
que puede efectuarse de z formas diferentes, entonces el número total de formas
diferentes que pueden efectuarse estas n acciones es igual con:

a x b x c x ... x z

Este principio también se llama principio del análisis combinatorio ó principio


multiplicativo.

Ejemplo 1: ¿De cuántas maneras diferentes podrá vestirse un joven que tiene 3
camisas diferentes, 4 pantalones y 2 pares de calzado?

Solución:

3 x 4 x 2 = 24 maneras diferentes

Ejemplo 2: En una ciudad los números de teléfono constan de 5 dígitos, cada uno de
los cuales se llama con alguno de los 10 dígitos (0 al 9). ¿Cuántos números diferentes
pueden formularse?

Solución:

69
10 x 10 x 10 x 10 x 10 = 100,000 números diferentes

Ejemplo 3:

La agencia de Publicidad PIPSA, ha obtenido la exclusividad respecto a una línea de


polvos para preparar postres. A estos efectos la agencia ha decidido organizar un
concurso nacional destinado a adivinar el nombre futuro de esa línea de productos.

Las condiciones son: 

a)      Los nombres que se propongan deben ser de 4 letras.

b)      Ninguna letra debe repetirse.

c)      La primera y tercera letras deben ser consonantes.

d)      La segunda y cuarta letras deben ser vocales.

e)      Si una persona propone 2 veces el mismo nombre queda descalificada.

 ¿Cuántos nombres debe proponer una persona para estar seguro que participa en el
sorteo público?

Considerar 28 letras del alfabeto


 Solución:

 23 x 5 x 22 x 4 = 10,120 nombres diferentes 

¿Por qué esos números? 

Porque hay 28 letras del alfabeto, 23 consonantes y 5 vocales, pero se disminuyó de


23 a 22 en la primera y tercera cifra porque una de las condiciones es que las letras no
se repitan. Así como 5 y 4 en la segunda y cuarta cifras, que son las vocales.

Notación Factorial
En algunos problemas de matemáticas se nos presentan multiplicaciones de números
naturales sucesivos tal como:

4 x 3 x 2 x 1 = 24 ; 3x2x1=6 ; 2 x 1 = 2.

Para abreviar estas expresiones, se usa una notación especial llamada notación
factorial y nos denota las multiplicaciones sucesivas de n hasta l y se define como:

70
 4 x 3 x 2 x 1 = 4!  Se lee: “cuatro factorial” o “factorial de cuatro”

3 x 2 x 1 = 3! Se lee: “tres factorial” o “factorial de tres”

 En términos generales:

n(n-1)(n-2)...x 2 x 1 = n! Se lee “n factorial” o “factorial de n”

Propiedades:

 a)      para n natural

n! = n(n-1)!

Ejemplo: 7! = 7 x 6! = 7 x 6 x 5 x 4!

b)      0! = 1

Ejemplos:

1)      5! = 5 x 4 x 3 x 2 x 1 = 120

2)      4! 3! = (24)(6) = 144

3)     

4)     

5)     

 Cuando n es demasiado grande se suele utilizar la fórmula de Stirling:

Ejemplo:

Determinar 50! por Stirling:

71
INTRODUCCIÓN A LA PROBABILIDAD

Azar y Desconocimiento.

El azar está relacionado con el desconocimiento. Un ejemplo nos puede


ayudar; piense en un proceso industrial que produce grandes cantidades de un
artículo determinado. No todos los artículos producidos son idénticos, cada
artículo puede calificarse como ``bueno'' o ``defectuoso''. Si de toda la
producción se escoge un artículo ``a ciegas'', ese artículo puede resultar bueno
o defectuoso. Esta es una situación azarosa (o aleatoria) y la parte esencial de
este azar es que no sabemos si el artículo seleccionado es defectuoso. Claro
que con experiencia en el proceso es posible cuantificar de una manera
numérica qué tan factible es que el artículo sea defectuoso o no.

Azar e incertidumbre.

Hay otro concepto asociado al azar y es el de incertidumbre. Veamos un


ejemplo. Respecto a una inversión, podemos estar contemplando invertir una
cantidad de dinero. El retorno sobre la inversión puede ser fijo, como en el caso
de una cuenta en un banco con interés fijo; pero pensemos en una empresa. El
negocio puede resultar desde un gran éxito hasta un fracaso, es decir, la
ganancia no es fija, sino que depende del éxito a obtener. Si no podemos
evaluar qué tan factible es cada monto posible de la ganancia, tenemos una
situación de incertidumbre. Por el contrario, si

podemos tener una idea de qué tan probables son los diferentes resultados y
entonces tendremos una situación de riesgo. Esta última es la que llamamos
aleatoria o azarosa.

72
Espacio Muestral y Probabilidad.

El párrafo anterior se resume diciendo que en las situaciones o experimentos


aleatorios tenemos dos elementos esenciales:

1. Una lista de posibilidades a futuro: espacio muestral


2. Una cuantificación de la incertidumbre sobre esa lista de posibilidades:
asignación de probabilidades.

Cualquier problema o situación en la probabilidad, parte de esos dos


elementos: Espacio Muestral y Probabilidades.

ESPACIO MUESTRAL.

El espacio muestral es el conjunto de todos los posibles resultados de un


experimento o situación aleatoria.

Si en una caja hay 10 manzanas y 2 están echadas a perder (¡al menos en


este momento!), al extraer tres manzanas y ver cuantas son buenas podemos
obtener 1, 2 o 3 buenas (¡0 buenas es imposible!). De modo que en este
ejemplo el espacio muestral es: { 1, 2, 3 }.

Si un juego consiste en tirar todos los volados que hagan falta hasta obtener
tres águilas seguidas o hasta que sean 15 volados, si nos fijamos en el número
de volados requeridos, el espacio muestral es: { 3, 4, 5, . . . , 15 }. Pero si nos
fijáramos en el número de soles que resultan, entonces el espacio muestral es:
{ 0, 1, 2, . . . , 15 }.

Es claro que para determinar el espacio muestral en un experimento aleatorio


es necesario entender perfectamente:

 Qué se va a hacer.
 Qué se va a observar o contar.

73
SUCESOS O EVENTOS.

Cuando se tiene un espacio muestral llamamos, formalmente evento a


cualquier subconjunto del espacio muestral.

Decimos que un evento se realiza, cuando el resultado del experimento


aleatorio es un elemento del evento.

Las dos definiciones anteriores son muy abstractas. Veamos un par de


ejemplos.

En el caso de contar cuantos volados hacen falta para conseguir tres águilas
seguidas o tirar 15 volados; el espacio muestral son los números: 3, 4, 5, . . . ,
15.

Un evento podría ser { 3, 5, 7, . . . , 15}.

Este evento corresponde a que el número de tiros necesario sea n ó n. Si al


hacer los volados los resultados fueran:

 AASAASSSAAA (aquí nos detenemos porque han caído ya, tres águilas
seguidas), el evento si se realizó porque el número necesario fue 11 y es n ó
n.
 SSSAAA (aquí paramos porque ya hay tres águilas), el evento no se realizó.

Podemos pensar que cada experimento al azar es un juego y que un evento es


una lista de los resultados que hacen que YO gane.

Otro ejemplo más. Al comprar llantas para mi auto, puede ser que manifiesten
un defecto de fabricación dentro del período de garantía total y que el
fabricante deba reponerlas. También puede pasar que el defecto se manifieste
en el período de garantía parcial y que el fabricante bonifique sólo un
porcentaje o que el defecto se manifieste después de vencido el período de
garantía en cuyo caso el fabricante no paga nada. También puede pasar que
las llantas no tengan defecto de fabricación aparente y que no haya garantía
que reclamar. Como se puede considerar que las llantas que me vendieron se

74
escogieron al azar de entre toda la producción, tenemos un experimento
aleatorio.

El espacio muestral en este experimento es: S = { T, P1, P2, P3, N, OK }. Con


la siguiente notación T: pago total, P1 pago del 50%, P2: pago del 30%, P3:
pago del 10%, N: nada de pago, OK: llantas sin defecto. El evento { OK } sólo
se realiza cuando las llantas no tienen defecto.

En este último ejemplo se tiene un evento simple porque consta de un solo


punto del espacio muestral. Será compuesto cuando tiene varios puntos del
espacio muestral. Se llama evento imposible al que no puede ocurrir; éste
evento corresponde al conjunto vacío. Otro evento extremo es el espacio
muestral mismo que, puesto que siempre ocurre, se llama evento seguro.

Problemas Propuestos:

1. En una caja hay 8 focos de los cuales 3 están fundidos. Se van a sacar los
focos de uno en uno, hasta encontrar los tres fundidos. Si nos fijamos en el
número de focos que se quedan en la caja ¿cuál es el espacio muestral?
2. En el experimento de los volados mencionado arriba. Si nos fijamos en el
número de soles que salieron, describa en sus propias palabras, cuál es el
evento { 0, 1, 2 }. Si los resultados fueron AASAASAAA ¿Por qué se detuvo el
experimento? ¿Se realizó el evento?
3. Júntese con un compañero de este curso y entre los dos discutan y encuentren
un ejemplo de un experimento aleatorio relacionado con las personas que
están en la biblioteca después de las 10 de la noche. Expliquen cuál es el
espacio muestral. Expliquen qué información necesitarían para asignar
probabilidades.
4. Con su mismo compañero, encuentren un ejemplo de un experimento aleatorio
referente a las inscripciones. Detallen el espacio muestral. Propongan un
evento. Den un ejemplo de un resultado que implique que el evento no se
realizó y otro resultado donde el evento sí se haya realizado.

75
76
SESION 05

PRACTICA CALIFICADA

SESION 06

PROBABILIDAD

Aparte del espacio muestral, en cada experimento aleatorio hay una asignación
primaria de probabilidades. Basados en la experiencia o en razonamientos de
simetría, a cada elemento del espacio muestral le asignamos una evaluación
de qué tan factible es. Esta evaluación se refleja en un porcentaje (número
entre 0 y 1). Entre más factible sea el resultado, mayor es el porcentaje que se
le asigna. Los casos extremos son:

 Un evento que no puede suceder, tiene probabilidad cero. Muchas veces estos
eventos con probabilidad cero son imposibles por alguna contradicción lógica
en su definición. Por ejemplo: ``que la suma de dos dados sea n ó n y los dos
dados tengan el mismo número''.
 En el otro extremo hay eventos que siempre suceden y estos tienen
probabilidad uno. Por ejemplo: ``que el número de águilas en dos volados sea
menor o igual a 7.8'', aunque el evento pueda resultar extraño en su definición,
siempre sucede y tiene probabilidad igual a 1.

La asignación toma la forma matemática de una función y se llama función de


probabilidad.

El dominio de esta función es el espacio muestral y su codominio es el intervalo


real [ 0, 1 ].

Esta función nos da las probabilidades de los eventos simples. Para un evento
compuesto, simplemente sumamos las probabilidades de los elementos que lo
componen.

77
EJEMPLOS.

La probabilidad es un concepto muy viejo en las matemáticas. Actualmente lo


aplicamos en prácticamente todos los campos de la actividad humana. La
variedad de situaciones en que se aplica va desde la producción hasta la
planeación a largo plazo de las empresas.

Aunque esta es la visión moderna, y la razón de que estemos estudiando


probabilidad en este curso, los orígenes del concepto son muy humildes,
comenzó aplicándose a los juegos de azar.

Pensemos en un dado perfectamente balanceado de modo que ninguno de los


seis lados es favorecido.

El espacio muestral es { 1, 2, 3, 4, 5, 6 }.

La función de probabilidad le asigna a cada uno de los elementos del espacio


muestral el valor de 1 / 6.

Esta asignación la hacemos porque el dado está balanceado.

Decimos que la probabilidad de un evento es el número de resultados


favorables al evento entre el número de resultados posibles.

Por ejemplo, la probabilidad de que el resultado sea mayor que 4 es 2 / 6,


porque hay 2 resultados favorables entre los 6 resultados posibles.
Formalmente, el evento es A = { 5, 6 } y P[A] = 2/6.

La probabilidad que resulta de esta manera, tiene una interpretación empírica;


si hacemos una serie larga de lanzamientos del dado, y observamos la
frecuencia de resultados favorables al evento A, esta frecuencia tiende a ser
2/6.

78
Otro ejemplo: una urna con 50 papelitos numerados de los cuales se escoge
uno para que tenga un premio. El espacio muestral es { 1, 2, 3, . . . , 50 }. La
asignación de probabilidades es de 1 / 50 para cada resultado. Si yo compré
los números 1, 14 y 18; el evento de que yo gane es { 1, 14, 18 } y la
probabilidad de que gane es 3 / 50.

SESION 07

PROBABILIDAD DE SUCESOS

Al definir los sucesos hablamos de las diferentes relaciones que pueden


guardar dos sucesos entre sí, así como de las posibles relaciones que se
pueden establecer entre los mismos. Vamos a ver ahora cómo se refleja esto
en el cálculo de probabilidades.

a) Un suceso puede estar contenido en otro: entonces, la probabilidad del


primer suceso será menor que la del suceso que lo contiene.

Ejemplo: lanzamos un dado y analizamos dos sucesos: a) que salga el número


6, y b) que salga un número par. Dijimos que el suceso a) está contenido en el
suceso b).

P(A) = 1/6 = 0,166

P(B) = 3 / 6 = 0,50

Por lo tanto, podemos ver que la probabilidad del suceso contenido, suceso a),
es menor que la probabilidad del suceso que lo contiene, suceso b).

b) Dos sucesos pueden ser iguales: en este caso, las probabilidades de


ambos sucesos son las mismas.

Ejemplo: lanzamos un dado al aire y analizamos dos sucesos: a) que salga


número par, y b) que salga múltiplo de 2. Las soluciones coinciden en ambos
casos.

P(A) = 3 / 6 = 0,50

P(B) = 3 / 6 = 0,50

c) Intersección de sucesos: es aquel suceso compuesto por los elementos


comunes de los dos o más sucesos que se intersectan. La probabilidad será
igual a la probabilidad de los elemntos comunes.

79
Ejemplo: lanzamos un dado al aire y analizamos dos sucesos: a) que salga
número par, y b) que sea mayor que 3. La intersección de estos dos sucesos
tiene dos elementos: el 4 y el 6.

Su probabilidad será por tanto:

P(A  B) = 2 / 6 = 0,33

SESION 08

PROBABILIDAD DE SUCESOS

d) Unión de dos o más sucesos: la probabilidad de la unión de dos sucesos


es igual a la suma de las probabilidades individuales de los dos sucesos que se
unen, menos la probabilidad del suceso intersección

Ejemplo: lanzamos un dado al aire y analizamos dos sucesos: a) que salga


número par, y b) que el resultado sea mayor que 3. El suceso unión estaría
formado por los siguientes resultados: el 2, el 4, el 5 y el 6.

P(A) = 3 / 6 = 0,50

P(B) = 3 / 6 = 0,50

P (A  B) = 2 / 6 = 0,33

Por lo tanto,

P (A u B) = (0,50 + 0,50) - 0,33 = 0,666

e) Sucesos incompatibles: la probabilidad de la unión de dos sucesos


incompatibles será igual a la suma de las probabilidades de cada uno de los
sucesos (ya que su intersección es el conjunto vacio y por lo tanto no hay que
restarle nada).

Ejemplo: lanzamos un dado al aire y analizamos dos sucesos: a) que salga un


número menor que 3, y b) que salga el número 6.

80
La probabilidad del suceso unión de estos dos sucesos será igual a:

P(A) = 2 / 6 = 0,333

P(B) = 1 / 6 = 0,166

Por lo tanto,

P(A u B) = 0,33 + 0,166 = 0,50

f) Sucesos complementarios: la probabilidad de un suceso complementario a


un suceso (A) es igual a 1 - P(A)

Ejemplo: lanzamos un dado al aire. el suceso (A) es que salga un número par, luego
su complementario, suceso (B), es que salga un número impar.

La probabilidad del suceso (A) es igual a :

P(A) = 3 / 6 = 0,50

Luego, la probabilidad del suceso (B) es igual a:

P(B) = 1 - P(A) = 1 - 0,50 = 0,50

Se puede comprobar aplicando la regla de "casos favorables / casos posibles":

P(B) = 3 / 6 = 0,50

g) Unión de sucesos complementarios: la probabilidad de la unión de dos


sucesos complementarios es igual a 1.

Ejemplo: seguimos con el ejemplo anterior: a) que salga un


número par, y b) que salga un número impar. La probabilidad del
suceso unión de estos dos sucesos será igual a:

P(A) = 3 / 6 = 0,50

P(B) = 3 / 6 = 0,50

Por lo tanto,

P(A U B) = 0,50 + 0,50 = 1

81
SESION 09

AXIOMAS DE LA PROBABILIDAD

Teniendo en cuenta las operaciones para hacer conjuntos nuevos, hay algunos
hechos fundamentales respecto a la probabilidad que se cumplen siempre:

1. P(A) mayor o igual a 0


2. P(S) = 1
3. P(A ó B) = P(A) + P(B) si A y B son excluyentes.

De estas tres propiedades, los matemáticos deducen un montón de reglas


útiles para calcular probabilidades en situaciones más complicadas. A este tipo
de proposiciones de las que se deducen otras, se les llama axiomas y los tres
de arriba son los axiomas de la probabilidad. Algunas de las fórmulas más
útiles, deducidas de los axiomas, son las siguientes.

 P( vacío ) = 0
 P(A') = 1- P(A)
 P(A - B) = P(A) - P(A y B)
 Si A está contenido en B entonces P(A) menor o igual a P(B)
 P(A) menor o igual a 1
 P(A ó B) = P(A) + P(B) - P(A y B).

La deducción de estas leyes a partir de los tres axiomas es un ejercicio de


ingenio matemático al que valdría la pena asomarse, pero en el que no
tenemos intención de meternos de lleno. Ya que desde el punto de vista de
este curso, lo interesante es aplicarlas.

Respecto a la tercera de la reglas, note bien que la resta de conjuntos se define


así: ``A - B'' es la colección de elementos de A que no están en B. De tal suerte
que P(A - B) debe contemplar sólo a elementos de A y por eso es que a P(A)
no le restamos P(B) sino solamente P(A y B).

Otro comentario lo merece la última regla:

82
P(A ó B) = P(A) + P(B) - P(A y B).

Es preciso restar P(A y B) ya que así no lo hiciéramos, se estaría tomando en


cuenta dos veces a los elementos comunes a A y a B.

Problemas Propuestos

Resolver los siguientes ejercicios

1. Inventen un juego con dos dados. Como ejemplos,


o tirar dos dados y el resultado no es la suma de los dados sino la
multiplicación.
o tirar dos dados y el resultado es 1000 si la suma de los dos es par y
5000 si la suma es n ó n.
Escriban el espacio muestral y, teniendo en cuenta que las 36 parejas de
posibles resultados con dos dados: { (1,1), (1,2), (1,3), . . . } son igualmente
probables, encuentren la función de probabilidad para el juego que inventaron.

Combinaciones de Sucesos o Eventos.

Definir la unión e intersección de eventos. Obtener la probabilidad de la unión e


intersección de eventos. Enunciar y aplicar las leyes de la probabilidad. Definir
eventos mutuamente excluyentes y la partición de un espacio muestral.
Establecer y aplicar la ley de la adición de la probabilidad para n eventos.

De la función de Probabilidad al cálculo de probabilidades

Toda la información matemáticamente importante respecto a un experimento


aleatorio se encuentra en:

 El espacio muestral.
 La función de probabilidad.

83
El cálculo de la probabilidad de un evento se simplifica partiéndolo en eventos
más sencillos y uniendo los pedazos de acuerdo a la llamada ley de la adición
para probabilidades.

Un ejemplo nos puede servir. Se van a tirar dos dados y yo gano si

 la suma de los dados da siete o


 aunque la suma no sea siete, si uno, al menos, de los dados cae en uno.

Los resultados de tirar los dados son 36: S = { (1,1),(1,2),(1,3), . . . ,(6,6) }.


Además, por la simetría interna de los dados, cada uno de estos 36 resultados
es igualmente probable. Esto establece la función de probabilidad.

Pasemos al problema de calcular la probabilidad de ganar. Una manera


equivocada de resolver el problema es así. Yo gano si

 el primer dado cae uno o


 el segundo dado cae uno o
 la suma de los dos es siete.

Como las respectivas probabilidades son: 1 / 6, 1 / 6 y 1 / 6. La probabilidad de


que gane es la suma de estas tres, 1 / 2. Lo que tiene mal este razonamiento
es que los eventos en que hemos partido el resultado de que yo gane no son
ajenos, y en estas circunstancias no se vale sumar las probabilidades y ya.

Para responder correctamente hay que partir el resultado de que yo gane en


más pedazos:

 que el primer dado caiga uno y el otro no o


 que el segundo caiga uno y el primero no o
 que los dos caigan uno o
 que la suma sea siete pero no haya ningún uno.

Las respectivas probabilidades son: 5 / 36, 5 / 36, 1 / 36 y 4 / 36. Para una


probabilidad total de ganar de: 15 / 36; esto es menor que 1 / 2 que es la que
habíamos calculado mal.

Fíjese que para resolver el problema lo partimos en pedazos más pequeños,

84
 los pedazos son ajenos.
 la probabilidad fue la suma de esos pedazos.

En el ejemplo usamos un espacio muestral equiprobable.

Problemas propuestos:
Resuelva estos ejercicios.

1. Se van a tirar 5 monedas y el resultado va a ser el número de águilas menos el


número de soles. Escriban el espacio muestral equiprobable para este
experimento. (Debe tener 32 resultados).
2. Siguiendo con el ejercicio anterior, me van a dar una cantidad de pesos igual a
la resta: [número de águilas] MENOS [número de soles]. Si sale negativo
quiere decir que ¡yo pago! ¿Cuál es la probabilidad de que gane más de dos
pesos?
3. Si un dado se construye de modo que un 1 o un 2 ocurran dos veces más
frecuentemente que un 5, mismo que se presenta tres veces más seguido que
un 3 o un 4 o un 6. ¿Cuál es la probabilidad de que el número que se obtiene
sea par? ¿Cuál la de que sea un cuadrado perfecto? ¿Cuál la de que sea
mayor que 4?
4. ¿Cómo harían Uds. para construir un dado como el que se propone en el
ejercicio anterior?

85
SESION 10

EXAMEN PARCIAL

SESION 11
PROBABILIDAD CONDICIONAL

Consideremos la siguiente situación. Se tienen tres urnas similares; por fuera


son idénticas. Se sabe que

 en la urna 1 hay 3 bolas blancas y 19 azules,


 en la urna 2 hay 20 bolas blancas y 2 azules,
 en la urna 3 hay 11 bolas blancas y 11 azules.
Se va a sacar una bola de una de las urnas. Puede ser azul o blanca. ¿Cuál es la
probabilidad de que sea blanca?

Hay cuatro posibles soluciones:

1. La probabilidad de una blanca es 3 / 22. Esto es porque si se escoge la urna 1,


hay 3 de 22 bolas que son blancas. Esta respuesta nos deja pensando en que
es muy arbitrario decir que la urna escogida es la 1. Si la urna escogida fuese
la 1 esta sería la respuesta correcta.
2. De manera similar, podemos pensar que la urna escogida es la 2 y entonces la
probabilidad de una bola blanca es 20 / 22.
3. Claro que, también, la urna escogida puede ser la 3 y entonces la probabilidad
de blanca es 11 / 22.
4. Como no se sabe cual es la urna escogida y las tres urnas tienen el mismo
número de bolas, la probabilidad se calcula como si fuese una gran urna con
66 bolas de las cuales 3 + 20 + 11 son blancas y, así, la probabilidad es 34 / 66
¿Cuál es la respuesta correcta? o ¿habrá otra que sea la respuesta correcta?

Una cosa es clara; si podemos suponer que la urna escogida es la 1, la


respuesta correcta es la primera. Lo mismo se puede decir de la segunda y la
tercera. La cuarta es un poquito más atrevida y quizá sea correcta. Por lo

86
pronto vamos a darle un nombre a las tres primeras: les llamamos probabilidad
condicional.

 A la primera la llamamos ``probabilidad condicional de blanca dado que la urna


es la 1''.
 A la segunda, la llamamos de manera similar condicional de blanca dado que la
urna es la 2.
 A la tercera se le da un nombre análogo [¿Cuál nombre?].
Más adelante en el curso, veremos lo que se llama fórmula de la probabilidad total y
entonces, veremos que la cuarta respuesta daría la ``probabilidad no condicional''.

Por el momento ampliemos nuestras ideas sobre probabilidad condicional con


un poco de matemáticas.

Formalmente, definimos en clase la probabilidad condicional de la siguiente


manera:

P( A | B ) = [P( A y B )] / [P( B )]

 El símbolo P( A | B ) lo leemos como probabilidad de A dado B. Lo


interpretamos como la probabilidad de que, sabiendo que ya sucedió B,
además suceda A. En el ejemplo de las urnas A sería el evento ``la bola es
blanca''; B sería la urna correspondiente.
 Como lo que está abajo en el quebrado es la probabilidad de lo dado, la
fórmula no es simétrica en A y B. Si los intercambiamos, da otro número. Esto
se ve en el ejemplo ya que no es lo mismo que nos informen cual es el número
de la urna escogida a que nos digan que la bola fue blanca y nos pregunten
cuál es la urna.
 Esta fórmula no tiene sentido matemático si P(B) = 0. En tal caso decimos que
la probabilidad condicional no está definida. Claro que eso está bien porque no
puede haber sucedido algo que es imposible.

Fíjese que esta fórmula se usará cuando haya una manera fácil de calcular las
probabilidades no condicionales y la condicional sea difícil. Eso no fue el caso
con el color de la bola y las urnas.

87
Para ejemplificar el tipo de situación en que nos sirve la fórmula descrita,
considere este problema.

Se tiran dos dados y se sabe que el primero no tiene el número 5. ¿Cuál es la


probabilidad de que la suma de los dados sea 8?

Para resolver, llamemos

 B el evento: ``el primer dado no es 5''.


 A el evento: ``la suma de los dados es 8''.
Con los datos se ve que:

 P(B) = 30 / 36.
Porque de las 36 parejas posibles, 6 tienen 5 en el primer dado.

 P(A y B) = 4 / 36.
Porque sólo se obtiene 8, con las parejas (2,6), (3,5), (4,4) y (6,2) [La pareja (5,3)
sí suma ocho pero tiene un 5 en el primer dado].

y, usando la fórmula, P(A|B) = 4 / 30.

También hubiéramos podido calcular sin la fórmula, pero esa cuenta requiere
más ingenio. En este ejemplo es fácil calcular las probabilidades no
condicionales.

Hay muchos problemas, como en el de las urnas, en que lo contrario es lo


cierto: es fácil calcular la condicional y la podemos usar para calcular la
conjunta.

Si despejamos a P(A y B), tendremos una fórmula para calcular la probabilidad


conjunta cuando sea fácil calcular la condicional.

En clase hacemos un ejemplo simple de cálculo de probabilidad condicional


con una tabla de dos clasificaciones cruzadas.

En ese ejemplo se ven tres cosas:

88
1. La probabilidad condicional nos permite medir la información. En los ejemplos
vimos como cambia la probabilidad de A, antes de conocer nada: P(A) y
después de conocer la ocurrencia de el evento B: P(A | B).
2. En un extremo está el cambio enorme que corresponde a que A y B sean
excluyentes (ajenos). En este caso la probabilidad podría llegar incluso a ser
cero.
3. En el otro extremo están los eventos en los que sucede que P(A | B) = P(A).
Esto quiere decir que la información de que B ocurrió no cambia la probabilidad
de A y decimos que A y B son independientes.

Esta última característica, la independencia, juega un papel muy importante en


la probabilidad y merece una atención más detallada. Por el momento debemos
establecer una definición:

A y B son eventos independientes si y sólo si P(A y B) = P(A) P(B)

En forma equivalente decimos:

A y B son eventos independientes si y sólo si P(A | B) = P(A)

La equivalencia se sigue de una sustitución algebraica muy sencilla.

La consecuencia de que esta sea una definición es que:

para comprobar la independencia de dos eventos es preciso hacer ver


que P(A y B) = P(A)P(B).

Es importante remarcar la diferencia de concepto entre eventos independientes


y eventos excluyentes o ajenos. En nuestro ejemplo se ve claramente que
ambos conceptos son antitéticos. El hecho de que dos eventos se excluyan
casi implica que no son independientes. La excepción se da en el caso
degenerado de que alguno de ellos (o los dos), sea imposible. En el habla
cotidiana, a veces, se confunden estos conceptos.

Note que si A es imposible; P(A) = 0. Además ``A y B'' también es imposible y


se tiene P(A y B) = P(A)P(B) ya que ambos lados de la igualdad valen cero .
Pero éste es el único caso en que dos eventos son ajenos e independientes a

89
la vez; en términos geométricos la idea de independencia se asemeja a la
perpendicularidad y la de ``ajenos'' al paralelismo.

Probabilidades de Intersecciones de Sucesos o Eventos.

Establecer y aplicar la ley general multiplicativa de la probabilidad para n


eventos.

Definir independencia de n eventos.

Dada una colección de eventos, determinar si son o no independientes.

Establecer y aplicar la ley particular multiplicativa de la probabilidad para n


eventos independientes.

Probabilidades conjuntas

Con:

 la definición que hicimos de probabilidad condicional y


 la definición de independencia podemos establecer igualdades que nos auxilien
para calcular la probabilidad de la ocurrencia simultánea de dos eventos.
1. Para dos eventos en general.
P(A y B) = P(A) P(B | A)

o Esta igualdad no es más que la definición de probabilidad condicional


volteada al revés.
o Para aplicar esta igualdad es preciso que contemos, de alguna manera
indirecta, con la probabilidad condicional.
o La igualdad se puede escribir también, condicionando sobre B, así.
P(A y B) = P(B) P(A | B)

2. Para dos eventos independientes


P(A y B) = P(A) P(B)

o Para poder usar esta igualdad se necesita saber, de otras fuentes, que
A y B son independientes.

90
o Esta igualdad no es más que la versión de la de arriba cuando P(B | A)
= P(B).

Estas igualdades son muy útiles cuando el experimento aleatorio se va a llevar


a cabo en etapas temporales. Por ejemplo, suponga que una empresa recibe
materia prima empaquetada en sobres de 300g. que vienen en cajas de 50
sobres cada una; suponga, además que cada sobre puede ser: bueno o
deficiente. Para revisar una caja, se van a tomar, al azar, 3 sobres.

 si más de 2 sobres son deficientes se rechazará la caja completa.


 si ningún sobre es deficiente, se aceptará la caja completa.
 si hay 1 o 2 sobres deficientes, se tomarán otros 3 sobres y si el total de
deficientes de los 6 sobres revisados, se pasa de 2, se rechazará la caja
completa (en caso de ser 2 o menos, se acepta la caja).
Una fuente secreta nos informa que una caja específica tiene 10 sobres deficientes.
¿Cuál es la probabilidad de que esa caja sea aceptada?

El problema es complejo, trate Ud. de resolverlo, la respuesta involucra pensar


en etapas, de acuerdo a los diferentes resultados de la primera etapa. El
problema lo resolvemos en el salón.

Más de dos Sucesos o Eventos

Ambas igualdades se pueden llevar a tres o más eventos, como sigue:

P(A y B y C) = P(A) P(B | A) P(C | A y B)

o cualquier otro orden para el condicionamiento,

por ejemplo:

P(A y B y C) = P(B) P(C | B) P(A | B y C)

Para el caso de eventos independientes la igualdad se simplifica en su


escritura:

P(A y B y C) = P(A) P(B) P(C)

91
La generalización de las fórmulas anteriores a más de tres eventos es
inmediata.

No olvide que para aplicar cualquiera de estas fórmulas es preciso conocer,


previamente los valores de las probabilidades involucradas.

Un ejemplo de uso de estas fórmulas es el siguiente:


Si una pistola está cargada con 15 cartuchos, de los cuales 2 son inútiles y no
funcionarán, ¿Qué probabilidad hay de que el primer cartucho funcione y los
dos siguientes no?

En este ejemplo, por la especificación del evento, podemos calcular las


probabilidades condicionales por separado y eso nos lleva aplicar la primera de
las fórmulas vistas arriba.

La solución la damos en el pizarrón.

Más sobre independencia

Respecto a la independencia de dos eventos, hay algunas cosas muy


elementales que agregar a la definición que hicimos en notas pasadas.

1. La independencia de dos eventos A y B, quiere decir que el saber que A


sucedió no modifica la probabilidad de que B también haya sucedido. Como
consecuencia saber que A no sucedió tampoco puede afectar a la probabilidad
de B.

Hacemos una demostración formal en el pizarrón.

Podemos poner esto diciendo que

Si A y B son independientes, también lo son las tres siguientes


pares: A' y B ; A y B' ; A' y B' (estamos usando el apóstrofe '
para denotar complemento)

2. Cuando se tienen tres eventos, se puede presentar una situación muy curiosa.
Puede pasar que
o A y B sean independientes y

92
o A y C sean independientes y
o B y C también sean independientes.
Pero A, B y C NO sean independientes.

Esta situación curiosa se describe diciendo que no basta que varios


eventos sean independientes a pares, para que sean independientes.

El ejemplo clásico es el de un experimento aleatorio con cuatro posibles


resultados igualmente probables: 1, 2, 3 y 4 .

o Si el resultado es 1, A gana y nadie más.


o Si el resultado es 2, B gana y nadie más.
o Si el resultado es 3, C gana y nadie más, pero
o Si el resultado es 4, los tres A, B y C ganan.
Usted puede calcular las probabilidades para darse cuenta que:

o P(A y B) = P(A) P(B)


o P(A y C) = P(A) P(C)
o P(B y C) = P(B) P(C)
pero P(A y B y C) no es igual a P(A) P(B) P(C).

3. Una nota final de un estilo menos matemático. La palabra independencia se


utiliza en otros contextos para denotar un sinnúmero de conceptos diferentes.

Los ejemplos más comunes son en política, en historia, en derecho. En


la ciencia se habla de variables independientes y el significado es
diferente que el que usamos aquí. Aún en otras ramas de la matemática
se usa la palabra independencia para denotar a otros conceptos.
Cuando queremos distinguir la definición técnica que usamos en la
probabilidad de otras nociones le ponemos un apellido a la
independencia y decimos independencia estocástica.

Es conveniente recordar que cuando existe duda si dos eventos son


independientes o nó, la única forma de zanjar la cuestión es viendo si P(A y B)
es igual o diferente al resultado de multiplicar P(A) P(B). Naturalmente que si la
independencia de dos eventos está en duda, el cálculo de P(A y B) no se

93
puede hacer simplemente multiplicando P(A) P(B) sino que se debe justificar de
alguna otra manera.

94
SESION 12
TEOREMA DE BAYES

Veamos un problema que nos llevará a una regla interesante de cálculo de


probabilidades que se llama: el teorema de Bayes.

En una etapa de la producción de un artículo se aplica soldadura y para eso se


usan tres diferentes robots. La probabilidad de que la soldadura sea defectuosa
varía para cada uno de los tres, así como la proporción de artículos que cada
uno procesa, de acuerdo a la siguiente tabla.

prob. prop.
robot
Defect. Proces.

A 0.002 18%

B 0.005 42%

C 0.001 40%

Tenemos un par de preguntas:

 Cuál es la proporción global de defectos producida por las tres máquinas


 Si tomo un artículo al azar y resulta con defectos en la soldadura, cuál es la
probabilidad de que haya sido soldado por el robot C.

(I) La primera pregunta nos va a llevar a lo que se conoce con el nombre de


fórmula de la probabilidad total.

Queremos conocer la proporción global de defectos delos tres robots. Después


de reflexionar un momento se ve que si todas las soldaduras las pusiera el
robot C, habría pocos defectos, serían 0.001 o 0.1%. En cambio, si todas las
pone el B, ¡sería un desastre!, tendríamos cinco veces más: 0.005 o 0.5%. De
modo que en nuestra respuesta debemos tener en cuenta las diferentes
proporciones de lo maquinado en cada robot.

95
Nuestra idea es empezar por descomponer el evento ``defectuoso'' en ``viene
del robot A y es defectuoso'' o ``viene del robot B y es defectuoso'' o ``viene del
robot C y es defectuoso''. En símbolos tendremos

P(d) = P(A y d) + P(B y d) + P(C y d)

P(d) = P(A) P( d|A) + P(B) P( d|B) + P(C) P( d|C)

Antes de ponerle números y resolver nuestro problema fijémonos en la fórmula


obtenida.

1. Hay tres eventos A, B y C que


o son ajenos y
o cubren todo el espacio muestral.
2. Conocemos las probabilidades de cada uno de ellos.
3. Además, conocemos las probabilidades condicionales de otro evento dado
cada uno de ellos.

La fórmula de arriba se llama fórmula de la probabilidad total.

Llenando con nuestros números, tenemos que

P(d) = (0.18)(0.002) + (0.42)(0.005) + (0.40)(0.001)

o sea que P(d) = 0.00286 casi 3 piezas por cada mil.

Es bueno comparar este resultado con los porcentajes de soldaduras


defectuosas de cada robot por separado. Podemos ver que el resultado se
encuentra entre todas ellas y se encuentra relativamente cerca de los
porcentajes de los robots más utilizados (el B y el C). Esto es muy razonable.

(II) La segunda pregunta es, a la vez más simple y más complicada. Nos va a
llevar a lo que se conoce con el nombre de teorema de Bayes.

La probabilidad que buscamos es una condicional pero al revés de las que


tenemos. Buscamos

96
P( C | d)

para calcularla usamos la definición de probabilidad condicional:

P( C | d) = [P(C y d)] / [P( d )]

El numerador (lo de arriba) lo calculamos con

P( C y d ) = P(C) P(d|C)

y el denominador lo calculamos con la fórmula de probabilidad total

P(d) = P(A) P( d|A) + P(B) P( d|B) + P(C) P( d|C)

juntando las dos tenemos la fórmula de Bayes:

P( C|d) = [P(C) P(d|C)] / [P(A) P( d|A) + P(B) P( d|B) + P(C) P( d|C)]

Aplicándola a nuestro caso tenemos

P(C|d) = [(0.40)(0.001)]/[(0.18)(0.002) + (0.42)(0.005) + (0.40)(0.001)]

o sea

P(C|d) = [0.0004]/[0.00286] = 0.1399

casi 14%.

O sea que si tomamos una pieza al azar, la probabilidad de que haya sido
soldada por el robot C es alta, 40%. Pero, como ese robot produce sólo 1 de
cada mil soldaduras defectuosas, al saber que la pieza seleccionada es
defectuosa, la probabilidad de que provenga del robot C disminuye a solamente
14%. Esto quiere decir que, en este caso el saber que la soldadura es
defectuosa, nos provee con una gran cantidad de información.

97
Si analizáramos, usando de nuevo la fórmula de Bayes las probabilidades de
los robots A y B, tendríamos

P(B|d) = 0.7343 y P(A|d) = 0.1259

Comparadas con las probabilidades de cada máquina sin saber que la pieza es
defectuosa vemos un gran incremento en la probabilidad de B.

Si, por el contrario la pieza no hubiese tenido defectos de soldadura, el mismo


teorema de Bayes nos daría (haga Ud. las cuentas y ¡fíjese que no me haya
equivocado yo!):

P(A|no d) = 0.1802 P(B|no d) = 0.4191 y P(C|no d) = 0.4007

Las probabilidades no son idénticas a las probabilidades no condicionales, pero


la diferencia es muy pequeña.

Para apreciar mejor el cambio, pongamos en una sola tabla las probabilidades
iniciales y las condicionales obtenidas bajo el conocimiento de la soldadura de
la pieza.

Robot P() P( |d) P( |no d)

0.125
A 0.18 0.1802
9

0.734
B 0.42 0.4191
3

0.139
C 0.40 0.4007
9

Es tan grande el éxito de los tres robots en el soldado correcto que el saber
que la pieza no tiene defectos, prácticamente no altera las probabilidades de
producción en uno u otro.

Por el contrario, el robot C es tan bueno, comparado con el B que, al saber que
la pieza es defectuosa, las probabilidades cambian dramáticamente.

98
En este ejemplo el cálculo de probabilidades condicionales nos cuantifica algo
que el sentido común nos dice de otra forma. Note que la fórmula de Bayes nos
sirvió para pasar de las probabilidades no condicionales a las condicionales.

Otro ejemplo del uso del teorema de Bayes

Otro ejemplo clásico del uso del teorema de Bayes es un problema de oro y
plata. Hay tres bolsas que tienen, cada una dos monedas. Las de la primera
son de oro, las de la segunda son de plata y las de la tercera son una de plata
y otra de oro. Se escoge una bolsa al azar y de ella una moneda también al
azar. Si la moneda es de oro, ¿cuál es la probabilidad de que la otra moneda
en la bolsa sea de oro también?

Primero notemos que la segunda bolsa no pudo haber sido elegida (porque no
tiene monedas de oro), sólo pudo haber sido seleccionada la primera o la
tercera. Si la bolsa elegida hubiese sido la tercera, el evento cuya probabilidad
nos interesa no se realiza. De modo que el evento que nos interesa es
equivalente a que se haya elegido la primera bolsa.

Una vez establecido lo anterior, apliquemos el teorema de Bayes para calcular

P(I|Au) = [P(I)P(Au|I)] / [P(I)P(Au|I) + P(II)P(Au|II) + P(III)P(Au|III)]

Las probabilidades que entran al lado derecho de la igualdad las sacamos,


inmediatamente, de las condiciones del problema y después de hacer cuentas
tenemos:

P(I|Au) = 2 / 3

Este problema es clásico porque existe una ``solución'' a la que muchas


personas llegan y es falsa. El argumento es el siguiente. Como todas las bolsas
son igualmente posibles, y el hecho de que la primer moneda extraída sea de
oro, nos indica que no se trata de la segunda bolsa. Concluimos que las dos
bolsas restantes tienen igual probabilidad y, por tanto, la probabilidad de que la
otra moneda sea de oro es 1/2.

99
Si Ud. piensa de acuerdo a este razonamiento (¡erróneo!), es muy difícil que
encuentre en qué se equivoca.

Lo que está mal es que lo que averiguamos, al saber que la moneda extraída
es de oro, es algo más que el rechazo de la segunda bolsa. Si sólo nos dijeran
que la bolsa escogida al azar no fue la segunda, sin informarnos del metal de la
moneda sacada, todavía tendríamos incertidumbre respecto a la primer
moneda; todavía podríamos apostar a si ésta es de oro o de plata. Al decirnos
que la moneda fue de oro, estamos aprendiendo algo más, y eso echa por
tierra el argumento de ``igual probabilidad para las dos bolsas restantes''.

Lo interesante del problema es que, si nos hubieran dicho que la moneda


sacada fue de plata, aplicando la fórmula de Bayes, llegamos a la conclusión
de que la probabilidad de que la otra moneda sea también de plata es 2/3
[¡Haga Ud. las cuentas!].

Es decir, si vamos a apostar al metal de la otra moneda, nos conviene apostar


por el metal de la primera.

Este ejemplo nos lleva a reflexionar sobre el uso adecuado de la información


contenida en ``lo dado'' en el cálculo de la probabilidad condicional.

Problemas Propuestos :

1. Una mujer portadora de hemofilia clásica da a luz tres hijos.

a) ¿Cual es la probabilidad de que de los tres hijos, ninguno esté afectado por
la enfermedad?

b) ¿Cual es la probabilidad de que exactamente dos de los tres niños esté


afectado?

2. El 60% de los individuos de una población están vacunados contra una cierta
enfermedad. Durante una epidemia se sabe que el 20% la ha contraído y que 2

100
de cada 100 individuos están vacunados y son enfermos. Calcular el porcentaje
de vacunados que enferma y el de vacunados entre los que están enfermos..

3. La proporción de alcohólicos que existe en la población de Málaga es,


aproximadamente, un 10%; no obstante, en las bajas que dan los médicos de
la Seguridad Social difícilmente se encuentra el diagnóstico de alcoholismo.
Aparecen sin embargo diagnosticados de hepatopatías, lumbalgias, etc., que
pueden hacer sospechar alcoholismo subyacente. Se realizó un estudio que
puso de manifiesto que el 85% de los individuos alcohólicos y el 7% de los no
alcohólicos sufrían tales patologías. Se desea saber cuál es la probabilidad de
que un individuo con esas patologías sea realmente alcohólico.

4. Dos tratamientos A y B curan una determinada enfermedad en el 20% y 30%


de los casos, respectivamente. Suponiendo que ambos actúan de modo
independiente, cuál de las dos siguientes estrategias utilizaría para curar a un
individuo con tal enfermedad:

a) Aplicar ambos tratamientos a la vez.

b) Aplicar primero el tratamiento B y, si no surte efecto, aplicar el A.

5. Se eligen al azar 3 deportistas de un equipo de 10 integrantes para realizar


un control antidopaje; Se sabe que 2 de los jugadores del equipo han tomado
sustancias prohibidas. ¿Cuál es la probabilidad de elegir para el análisis a
alguno de los infractores?

6. Estamos interesados en saber cuál de dos análisis A y B es mejor para el


diagnóstico de una determinada enfermedad, de la cual sabemos que la
presentan un 10% de individuos de la población. El porcentaje de resultados
falsos positivos del análisis A es del 15% y el de B es del 22%. El porcentaje de
falsos negativos de A es del 7% y de B es del 3%. ¿Cuál es la probabilidad de
acertar en el diagnóstico con cada método?

7. Con objeto de diagnosticar la colelitiasis se usan los ultrasonidos. Tal técnica


tiene una sensibilidad del 91% y una especificidad del 98%. En la población
que nos ocupa la probabilidad de colelitiasis es del 20%.

101
a) Si a un individuo de tal población se le aplican los ultrasonidos y dan
positivos, ¿cuál es la probabilidad de que sufra la colelitiasis?

b) Si el resultado fuese negativo, ¿cuál es la probabilidad de que no tenga la


enfermedad?

8. Entre los estudiantes de una Facultad de Filosofía y Letras se dan las


siguientes proporciones: el 40% son hombres. El 70% de los varones fuman,
mientras que entre las mujeres sólo fuman el 20%. Escogido un estudiante al
azar, calcúlese la probabilidad de que fume.

9. Los estudios epidemiológicos indican que el 20% de los ancianos sufren un


deterioro neuropsicológico. Sabemos que la tomografía axial computerizada
(TAC) es capaz de detectar este trastorno en el 80% de los que lo sufren, pero
que también da un 3% de falsos positivos entre personas sanas. Si tomamos
un anciano al azar y da positivo en el TAC, ¿cuál es la probabilidad de que esté
realmente enfermo?

10. Sabemos que tiene estudios superiores el 15% de la población española,


estudios medios el 40%, estudios primarios el 35% y no tiene estudios el 10%.
Los desempleados no se distribuyen proporcionalmente entre esas categorías,
dado que de entre los de estudios superiores están sin trabajo el 10%, entre los
de estudios medios el 35%, entre los de estudios primarios el 18%, y entre los
que no tienen estudios el 37%. Obtenga las probabilidades de que extraído uno
al azar, éste sea:

a) Titulado superior, sabiendo que está parado.

b) Un sujeto sin estudios que está en paro.

c) Un sujeto con estudios primarios o que está trabajando.

11. Una enfermedad puede estar producida por tres virus A, B, y C. En el


laboratorio hay 3 tubos de ensayo con el virus A, 2 tubos con el virus B y 5
tubos con el virus C. La probabilidad de que el virus A produzca la enfermedad
es de 1/3, que la produzca B es de 2/3 y que la produzca el virus C es de 1/7.

102
Se inocula un virus a un animal y contrae la enfermedad. ¿Cuál es la
probabilidad de que el virus que se inocule sea el C?

12. El 70% de los estudiantes aprueba una asignatura A y un 60% aprueba otra
asignatura B. Sabemos, además, que un 35% del total aprueba ambas. Elegido
un estudiante al azar, calcular las probabilidades de las siguientes situaciones:

a) Haya aprobado la asignatura B, sabiendo que ha aprobado la A.

b) Haya aprobado la asignatura B, sabiendo que no ha aprobado la A.

c) No haya aprobado la asignatura B, sabiendo que ha aprobado la A.

d) No haya aprobado la asignatura B, sabiendo que no ha aprobado la A.

13. La cuarta parte de los conductores de coche son mujeres. La probabilidad


de que una mujer sufra un accidente en un año es de 5/10.000, y para los
hombres es de 1/10.000. Calcúlese la probabilidad de que si acaece un
accidente, el accidentado sea hombre.

14. En un campus universitario existen 3 carreras sanitarias. Se sabe que el


50% cursan estudios de Enfermería, el 30% Medicina y el 20% Veterinaria. Los
que finalizaron sus estudios son el 20, 10 y 5% respectivamente. Elegido un
estudiante al azar, hállese la probabilidad de que haya acabado la carrera.

103
UNIDAD III

104
DISTRIBUCIONES DISCRETAS

Distribución de probabilidad

Distribución Binomial

En estadística, la distribución binomial es una distribución de probabilidad


discreta que cuenta el número de éxitos en una secuencia de n ensayos de
Bernoulli independientes entre sí, con una probabilidad fija p de ocurrencia del
éxito entre los ensayos. Un experimento de Bernoulli se caracteriza por ser
dicotómico, esto es, sólo son posibles dos resultados. A uno de estos se
denomina éxito y tiene una probabilidad de ocurrencia p y al otro, fracaso, con
una probabilidad q = 1 - p. En la distribución binomial el anterior experimento se
repite n veces, de forma independiente, y se trata de calcular la probabilidad de
un determinado número de éxitos. Para n = 1, la binomial se convierte, de
hecho, en una distribución de Bernoulli.

Para representar que una variable aleatoria X sigue una distribución binomial
de parámetros n y p, se escribe:

X ~ B(n, p)

La distribución binomial es la base del test binomial de significación estadística.

DISTRIBUCIÓN BINOMIAL

Se tiene un número fijo de pruebas n. Con las siguientes características:

 Cada prueba tiene sólo dos posibles resultados: genéricamente los llamamos
éxito y fracaso. Los denotamos con 1 (éxito) y 0 (fracaso).
 El resultado de cada prueba es independiente del resultado de las demás
pruebas.
 La probabilidad de éxito no cambia de una prueba a otra.

105
 Nos interesa sólo el número total de éxitos X y no el orden en que hayan
ocurrido.

Cuando se cumplen las condiciones anteriores X tiene la distribución binomial


con parámetros n y p, donde n es el número de intentos y p la probabilidad de
obtener un éxito. Los valores posibles son desde cero hasta ene: { 0, 1, 2, ... , n
}. La función de probabilidad es:

f(k) = P(X=k) = nCk pkq(n-k), para k en {0,1,2,...,n}

donde me he visto obligado por la tipografía, a usar el símbolo poco usual nCk para
denotar las combinaciones de k objetos tomados de un total de n:

Ck = [n!] / [k!(n-k)!]
n

además, la letra q representa la probabilidad de fracaso q = 1-p.

La media de la binomial es: E(X) = np y la varianza: var(X) = npq.

¿Dónde se aplica el modelo binomial?:

1. ¿En una familia de tres hijos, cuál es la probabilidad de que a lo más 2 sean
niñas? La probabilidad de niña es 0.5; el sexo de cada hijo es independiente
del de los demás; n=3 y p=0.5.
P(X <= 2) = P(X=0) + P(X=1) + P(X=2)
= 0.125 + 3(0.125) + 3(0.125)
= 0.875

o bien. P(X <= 2) = 1 - P(X=3) = 1 - 0.125 = 0.875

2. Se extraen 4 canicas CON REEMPLAZO de una urna que tiene 5 blancas y 3


negras. ¿Cuál es la probabilidad de que salgan menos de 2 blancas? Como la
elección se hace con reemplazo, se cumplen los requisitos de un experimento
binomial. X es el número de canicas blancas, n = 4 y p = 5/8 = 0.625. La
probabilidad que necesitamos es P(X < 2) y ésta es igual a:
P(X < 2) = P(X=0) + P(X=1)
= 0.3754 + (4)(0.625)(0.375)3
= 0.1516

106
3. De un lote con 1,000 artículos de los cuales el 10% son defectuosos, se
escogen al azar 10. ¿Cuál es la probabilidad de que haya más de 2
defectuosos? Aunque el muestreo se hace sin reemplazo, por la gran cantidad
de artículos que hay en el lote comparados con los que se escogen,
suponemos que son válidos los supuestos del modelo binomial. X es el número
de artículos defectuosos y la probabilidad que se nos pide es: P(X > 2).

Como el cálculo de las 8 probabilidades necesarias es muy latoso,


notemos que P(X > 2) = 1 - P(X <= 2). Aún así, hagamos esta cuenta
consultando la tabla de la binomial adecuada: n = 10, p = 0.10. La tabla
dice que P(X <= 2) = 0.9298. Por tanto, la probabilidad buscada es P(X >
2) = 0.0702 [Vea su tabla y descubra este valor]. Claro que también
tenemos la posibilidad de calcular P(X <= 2) y restarla de uno.

P(X <= 2) = P(X=0) + P(X=1) + P(X=2)


= (.9)10 + (10)(.1)(.9)9 + (45)(.1)2(.9)8
= 0.9298

Distribución Normal

Una distribución normal de media μ y desviación típica σ se designa por N(μ,


σ). Su gráfica es la campana de Gauss:

107
La media está representada por un triángulo y se puede interpretar como un
punto de equilibrio. Al arrastrarlo se modifica también la media. El mismo efecto
tiene el mover el punto correspondiente en la cúspide de la curva.

Ejercicios.

1.-Las puntuaciones de un examen se distribuyen normalmente con media 15


puntos. La puntuación A ha

sido superada por un 23% de los alumnos. La puntuación B está situada a 5


puntos diferenciales por

debajo de la media. Entre B y la media se encuentra el 30% de los alumnos.


Calcular :

a) La desviación típica de las notas.

b) Las puntuaciones directas de A y B.

a)

La puntución B=10, deja a su izquierda un área 0’20. Consultando

las tablas obtenemos un valor z 0'84. De aquí :

108
z = -0’84 = 10-15/s = -5/s→-5(-0'85)= 5'95

La distribución normal

De entre todas las distribuciones continuas tiene especial relevancia la distribución Normal o
de Gauss. Aparece frecuentemente en las situaciones más variadas.

Las variables que presentan una distribución Normal tienen características comunes tales
como la acumulación de valores en torno al valor de la media, la simetría en la distribución de
los valores y escasos valores alejados de la media, por ejemplo:

- Caracteres morfológicos de individuos: altura, peso, número de pie, tamaño del palmo,
etc.

- Carácterísticas de la mayoría de los productos de consumo: duración de las bombillas,


resistencia a la rotura de muebles o de piezas, duración de los electrodomésticos, etc.

- Calificaciones obtenidas en cursos, asignaturas y exámenes.

Se dice que una variable aleatoria continua sigue una distribución normal de media  y
X  N   , 
desviación típica  , y se escribe , cuando tiene la función de densidad:
2
−1 x− μ

f (x )=
1
e
( )
2 σ
σ √2 π

La gráfica de esta función de densidad tiene forma campaniforme, y se denomina “campana


de Gauss”.

Las Propiedades de la función f(x) se aprecian en su gráfica y son:

- f(x) tiene por dominio (−∞,+∞) .

- f(x) es continua en su dominio.

- f(x) es simétrica respecto a la recta x=μ.

109
1
(μ , )
- Ffx) tiene un máximo absoluto en σ √2 π
- f(x) tiene dos puntos de inflexión en x= μ+σ y x=μ-σ.

- f(x) es siempre positiva y asintótica con respecto al eje OX.

- La gráfica de la función de densidad f(x) se llama campana de Gauss .

Para el cálculo de probabilidades usamos la función de distribución:

x x −1 t−μ 2
F( X )=P( X≤x)= ∫ f (t )dt=
1 2 ( ) dt
.
σ
−∞
∫e
σ √ 2 π −∞ .

Para x=a este valor representa la probabilidad de que la v.a.X tome valores menores o
iguales que a y graficamente representa el área encerrada bajo la curva, el eje OX y la recta
x=a.

Puede observarse la dificultad de la integral, es por ello y dado lo habitual que es el uso de
esta distribución, que se utiliza una tabla ya confeccionada para el calculo de
probabilidades.

Pero como bien estarás pensando es imposible realizar una tabla para cada valor de μ y de
σ que pueden tomar los parámetros en la distribución.

Las curvas de las diferentes funciones de densidad son en realidad la misma curva variando
su máximo y su curvatura en función de μ y σ, por esto el área encerrada bajo la curva es
siempre la misma (como función de densidad que es vale 1) aunque repartida de forma
diferente.

Los anteriores argumentos justifican el uso de la N(0,1) como distribución estandar, para la
cual existe la tabla de valores de la función de distribución. A partir de ella y mediante un
cambio de variable, que denominamos tipificación podemos calcular las probabilidades
para cualquier distribución N(μ,σ)

110
Uso de tablas

La distribución N(0,1), recibe el nombre de distribución Normal reducida o estandar. Tiene de


media µ =0 y de desviación típica σ = 1. Su función de densidad es de la forma:

− x2
1 2
f (x )= e
√2 π

Las tablas nos ofrecen el valor de F(z0)= P(Z≤ z0) para valores de z0 de 0 a 4 . La primera
columna corresponde a valores desde 0,0 a 3,9 y el segundo decimal se completa con los
valores de la primera fila que van desde 0,00 a 0,09. En la intersección de la fila y la
columna correspondiente se encuentra el valor de dicha probabilidad.

Caso 1: P(Z≤ 1,56) = 0,9406

Buscamos el valor en la intersección de la fila de 1,5 y la columna 0,06.

La probabilidad pedida es el área sombreada

Caso 2: P(Z≥1,56) = 1 – P(Z≤ 1,56) =1 - 0,9406 = 0,0594

111
Tenemos en cuenta que el àrea total encerrada por la curva es 1 y procedemos por “paso
al contrario”

Caso 3: P(Z≤ -1,56) Habrás observado que en tabla sólo aparecen valores positivos de
la variable. Tenemos en cuenta que la función y sus valores son simétricos y por tanto

P(Z≤-1,56)=P(Z≥1,56) = 1 – P(Z≤1,56)=1- 0,9406 = 0,0594

Caso 4: P(Z≥ - 1,56) = P(Z≤ 1,56) = 0,9406

Utilizamos en nuestro razonamiento la simetria de la función

Caso 5:P(0,48≤ z ≤ 1,56) =P(Z≤1,56)-P(z≤0,48) = 0,9406 – 0, 6844 = 0,2562

Caso 6:P(- 0,48 ≤Z≤ 1,56) = P(Z≤1,56) - P(Z≤ - 0,48) = P( Z≤ 1,56) – P(Z≥
0,48) =P(Z≤ 1,56) – (1- P(Z≤ 0,48) ) =P(Z≤1,56) + P(Z≤ 0,48) -1
= 0.9406 + 0,6844 – 1 = 0,625

En ocasiones el problema planteado será el inverso, conocida la


probabilidad calcular los valores de la variable.

Caso 7:Hallar z0 si P(Z≤ z0) = 0, 9410

El valor no se encuentra en la tabla, pero el más aproximado es 0,9406.

Como 0,9406 esta en la intersección de la fila 1,5 y la columna 0,06 se


trata de z0 = 1,56

112
Caso 8: Hallar z0 si P(Z≤ z0) = 0,0594

Como 0,0594 no esta en la tabla, tampoco uno aproximado, deducimos


que z0 corresponde a un valor negativo y recordamos P(Z≤ -z 0) = 1 –
P(Z≤ z0) por tanto P(Z≤-z0) = 1- 0,0594 = 0,9406 que si aparece en la
tabla como –z0 = - 1,56.

Caso 9:Halla z0 si P( 0,48 ≤ Z ≤ z0) = 0, 2562

Como P( 0,48 ≤ Z ≤ z0) = P(Z≤z0) – P(Z≤ 0,48) = 0,2562 buscando en


la tabla P(Z≤z0) – 0,6844 = 0,2562 entonces despejando P(Z≤ zz) =
0,2562 + 0,6844 = 0,9406

Tenemos que P(Z≤ zz) = 0,9406

Tipificación de la variable

La tipificación consiste en pasar de la v.a. X con distribución N(μ,σ) a una nueva v.a. Z con
distribución N(0,1) mediante la expresión
x
Z

Sea X una v.a. con distribución N(60, 5) calcular:

a) P(X<70)
70−60
z 0= =2
Para x0 = 70 su correspondiente valor tipificado es 5
entonces segun la tabla de N(0,1) , P(X<70) = P(Z<2) = 0,9772

b) P(X > 75)


75−60
z 0= =3
Para x0 = 75 su correspondiente valor tipificado es 5
entonces según la tabla de N(0,1) ,P(Z>3) = 1-P(Z<3) = 1 – 0,9987 =
0,0013

113
c) P(70<X<75)

Utilizando las tipificaciones anteriores

P(70 < X < 75) =P(2 < Z < 3) = P(Z< 3) – P(Z< 2 ) = 0,9987 – 0,9772
= 0,0215

Distribución de probabilidades bajo la curva normal

Para cualquier distribución N( μ, σ ) se cumple que

P( μ – σ < X < μ+σ) = P(-1 < Z < 1) =


0,6826
La zona sombreada representa el 68,26% del área total.

La zona sombreada representa el 95,44% del área total P( μ-2σ < X < μ +2σ ) = P(-2 < Z < 2) =
0,9544

La zona sombreada representa el 99,74% del área total P( μ-3σ< X<μ +3σ ) = P(-3 < Z < 3) =
0,9974

Aproximación de la binomial a la normal


Aunque la binomial es una v.a. discreta y la normal es una v.a. continua
se puede observar que para cualquier valor de p en la binomial, al
aumentar los valores de n los polígonos de frecuencias asociados a los
diagramas de barras que represental la distribución van formando una
gráfica similar a la de la curva de densidad de una distribución normal.

114
Además cuando n es muy grande los cálculo en la distribución binomial
se vuelven muy complicados y con muchos sumandos.

Teorema de Moivre

En una distribución Binomial para valores de n grandes, n>30, y


cuando np≥5 y nq≥5 la distribución Binomial se puede aproximar
mediante una distribución Normal.

X es B(n,p) X´es N ( μ=np,σ=√ npq)

Por el teorema de Moivre podemos afirmar que la normal constituye


una muy buena aproximación de la binomial.

Para poder aplicar este cambio en la practica no podemos obviar que X


es una variable discreta y X´es una variable continua y que debemos
aplicar lo que se conoce como corrección de continuidad.

115
b)

La puntución A, deja a su izquierda un área

Consultando las tablas obtenemos un valor z = 0’74. De aquí :

z=0’74= A-15/ 5'95→A= 0’74 .5'95 + 15 =20'21

(El valor B=10 ya se determinó)

2.- Las puntuaciones de 1000 personas en un determinado test se distribuyen


normalmente. Sea X1 la

116
puntuación directa que supera el 84’13% de la distribución y X2 la puntuación
directa que es superada

por el 84’13% de la distribución. Sabiendo que X1 - X2 = 20, calcular :

a) Número de observaciones comprendidas entre las puntuaciones típicas 1’5 y


-0’2.

b) La desviación típica de la distribución.

a)

Pr (-0’2 < z < 1’5) = 0’93319 - 0’42074= 0’51245

Hay 1000 x 0’51245 = 512’45 ≈ 512 observaciones.

b)

⎧X = x2 +10

⎩X= x1 -10

117
Tablas : z = 1 deja a su izquierda un área 0’8413 :

z = 1 = x1 – X/s = x1-(x1-10)/s = 10/s

⇒ s=10

118

Potrebbero piacerti anche